You are on page 1of 46

Esophagus, stomach & duodenum

1. Which of the following statements about the anatomic course of the esophagus is correct?
A. The cervical esophagus passes behind and to the right of the trachea.
B. The thoracic esophagus enters the posterior mediastinum anterior to the aortic arch.
C. The thoracic esophagus passes behind the right mainstem bronchus and the pericardium.
D. The esophagus enters the diaphragmatic hiatus at the level of T8.
E. The esophagus deviates anteriorly and to the left as it enters the abdomen.
Answer: E

DISCUSSION: Knowledge of the normal course of the esophagus is important in operative and endoscopic procedures. The
cervical esophagus lies just anterior to the prevertebral fascia and courses through the neck posterior to the trachea and to
the left of the midline. The cervical esophagus is therefore more readily approached surgically through a left neck incision.
The thoracic esophagus enters the posterior mediastinum posterior to the aortic arch. Therefore, when operating on the
upper esophagus, a right thoracotomy is preferred, since the aortic arch overlies the esophagus in the left chest. The
esophagus descends in the posterior mediastinum behind the left mainstem bronchus and pericardium. It is the proximity of
the esophagus and left mainstem bronchus that is responsible for the development of malignant tracheoesophageal fistulas
between these two structures. The esophagus enters the diaphragmatic hiatus, which is located at the level of T11. As it
enters the abdomen, the esophagus deviates anteriorly and to the left. Therefore, when performing rigid esophagoscopy as
the distal esophagus is entered, the patient's head must be turned to the right and the esophagoscope elevated anteriorly to
avoid perforation.

2. Which of the following statements about esophageal anatomy is correct?


A. The esophagus has a poor blood supply, which is segmental in distribution and accounts for the high incidence of
anastomotic leakage.
B. The esophageal serosa consists of a thin layer of fibroareolar tissue.
C. The esophagus has two distinct muscle layers, an outer, longitudinal one and an inner, circular one, which are striated in
the upper third and smooth in the distal two thirds.
D. Injury to the recurrent laryngeal nerve results in vocal cord dysfunction but does not affect swallowing.
E. The lymphatic drainage of the esophagus is relatively sparse, localized primarily to adjacent paraesophageal lymph
nodes.
Answer: C

DISCUSSION: Poor technique, not poor blood supply, explains most esophageal anastomotic leaks. While the major blood
supply of the esophagus is from four to six segmental aortic esophageal arteries, there are extensive submucosal collaterals
from the inferior thyroid, intercostal, bronchial, inferior phrenic, and left gastric arteries. The esophagus lacks serosa and
instead is surrounded by mediastinal connective tissue (adventitia). There are two muscle layers in the esophagus, an outer
longitudinal and an inner circular one. Both layers of the upper third of the esophagus consist of striated muscle, while in
the lower two thirds they are (nonstriated) smooth muscle. The recurrent laryngeal branches of the vagus nerves provide
both parasympathetic innervation to the cervical esophagus and innervation to the upper esophageal sphincter (UES). Injury
to the recurrent laryngeal nerve therefore results in improved UES function with secondary aspiration on swallowing as well
as vocal cord dysfunction and hoarseness. The esophagus has extensive lymphatic drainage, with lymphatic capillaries
coursing longitudinally in the esophageal wall and communicating with paraesophageal, paratracheal and subcarinal, other
mediastinal, perigastric, and internal jugular lymph nodes. This accounts for the biologically aggressive nature of
esophageal carcinoma, which tends to metastasize early in its course.

3. Which of the following statements about the lower esophageal sphincter (LES) mechanism, or high-pressure zone
(HPZ), is true?
A. The LES is a circular smooth muscle ring that is 3 to 5 cm. long.
B. In assessing esophageal manometric data, mean HPZ pressure less than 6 mm. Hg or overall length less than 2 cm. is
more likely to be associated with incompetence of the LES and gastroesophageal reflux.
C. Esophageal manometry and the acid perfusion (Bernstein) test reliably identify the patient with an incompetent LES
mechanism.
D. Distal HPZ relaxation occurs within 5 to 8 seconds of initiating a swallow.
E. Twenty-four–hour distal esophageal pH monitoring is achieved with an intraesophageal pH electrode positioned at the
esophagogastric junction.
Answer: B

Asir Surgery MCQs Bank. © 1422H-2002- first impression ©


157
This project was raised after an idia by Dr. Gharama Al-Shehri (consultant surgeon). Developed and typed by Dr. Ghazi Al-Shumrani (intern).
Esophagus, stomach & duodenum
DISCUSSION: The LES is not an anatomic muscular sphincter like the anus or pylorus, but rather, is a functional sphincter
that serves as a barrier against abnormal regurgitation of gastric contents into the esophagus and is more appropriately
termed the distal esophageal HPZ. While, in general, no HPZ value absolutely indicates either competence or incompetence
of the LES mechanism, patients with a mean HPZ pressure less than 6 mm. Hg or a sphincter length less than 2 cm. are
likely to have an incompetent LES and gastroesophageal reflux. Esophageal manometry defines the amplitude and length of
the distal HPZ and the character of esophageal peristalsis. It does not determine whether or not the LES is competent. The
Bernstein test identifies the patient with an acid-sensitive esophagus but does not indicate whether or not the patient has
gastroesophageal reflux or esophagitis. Reflex distal HPZ relaxation occurs within 1.5 to 2.5 seconds after a swallow is
initiated and lasts 4 to 6 seconds. Twenty-four–hour distal esophageal pH monitoring is achieved by means of an
intraesophageal pH electrode positioned 5 cm. proximal to the HPZ, as determined by prior manometric evaluation.

4. Which of the following statements about esophageal motility is/are true?


A. The act of swallowing initiates UES relaxation, which persists until the bolus of food passes the LES.
B. The primary peristaltic wave normally propels the swallowed bolus through the esophagus in 4 to 8 seconds.
C. Normally, a progressive peristaltic contraction (primary wave) follows 50% of all swallows, the remainder being
secondary or tertiary contractions.
D. Secondary peristalsis is initiated when the entire swallowed bolus of food fails to empty from the esophagus into the
stomach.
E. Tertiary esophageal contractions are high-amplitude progressive peristaltic contractions that produce the “corkscrew”
appearance of esophageal spasm on barium esophagography.
Answer: BD

DISCUSSION: Swallowing initiates UES relaxation, which lasts only 0.5 to 1.0 second, considerably less than the 4 to 8
seconds required for a swallowed bolus to pass through the esophagus and into the stomach. A progressive peristaltic
contraction normally follows 97% of all swallows. If the swallowed bolus does not empty completely into the stomach,
distention of the esophagus initiates secondary peristalsis, which is progressive and sequential and persists until the retained
intraesophageal contents have passed into the stomach. Tertiary contractions are simultaneous, nonprogressive,
incoordinated contractions of the esophageal smooth muscle that produce the contraction rings responsible for the
corkscrew esophagus of diffuse esophageal spasm on barium esophagography.

5. Which of the following statements about UES dysfunction are correct?


A. This condition is diagnosed by the characteristic manometric findings of UES spasm.
B. Typical symptoms include cervical dysphagia, expectoration of saliva, and hoarseness.
C. The classic finding on barium esophagogram is a posterior cricopharyngeal bar.
D. Medical or surgical therapy of gastroesophageal reflux may be curative.
E. A cervical esophagomyotomy for UES dysfunction should be limited to 2 to 3 cm. in length so that normal muscle is not
damaged.
Answer: BCD

DISCUSSION: The unique anatomic characteristics of the UES and the limitations of existing equipment in recording the
rapid sequence of events associated with swallowing make standard manometric definition of UES motor abnormalities
extremely difficult. Characteristic consistent abnormalities of UES function in patients with cricopharyngeal dysfunction
have not been well documented. UES dysfunction results in cervical dysphagia, expectoration of saliva that is no longer
swallowed freely, and, often, intermittent hoarseness due to alteration of the larynx and vocal cords by the pull of the
abnormal cricopharyngeal sphincter. Barium esophagography in the patient with UES dysfunction frequently shows a
posterior cricopharyngeal “bar,” representing the prominent impression of the sphincter on the esophageal lumen. Patients
with gastroesophageal reflux may present with cervical dysphagia due to secondary UES dysfunction. Successful medical or
surgical treatment of the reflux may eliminate the cervical complaints. Since the UES is normally 3 to 5 cm. long, when a
cervical esophagomyotomy is required for treatment of cricopharyngeal dysfunction, a generous myotomy, 7 to 10 cm. long,
is carried out to ensure complete division of all incoordinated UES muscle fibers.

6. Which of the following statements about achalasia is/are correct?


A. In most cases in North America the cause is a parasitic infestation by Trypanosoma cruzi.
B. Chest pain and regurgitation are the usual symptoms.
C. Distal-third esophageal adenocarcinomas may occur in as many as 20% of patients within 10 years of diagnosis.
Asir Surgery MCQs Bank. © 1422H-2002- first impression ©
158
This project was raised after an idia by Dr. Gharama Al-Shehri (consultant surgeon). Developed and typed by Dr. Ghazi Al-Shumrani (intern).
Esophagus, stomach & duodenum
D. Manometry demonstrates failure of LES relaxation on swallowing and absent or weak simultaneous contractions in the
esophageal body after swallowing.
E. Endoscopic botulinum toxin injection of the LES, pneumatic dilatation, and esophagomyotomy provide highly effective
curative therapy for achalasia.
Answer: D

DISCUSSION: While in South America achalasia is the result of Chagas' disease caused by parasitic infestation by the
leishmanial forms of T. cruzi, in Europe and North America the cause of achalasia is unknown. The common presenting
symptoms of achalasia are dysphagia, regurgitation, and weight loss. Chest pain is an infrequent symptom in achalasia and
is more characteristic of esophageal spasm. Achalasia is a premalignant esophageal lesion: the retention esophagitis leads to
metaplasia and squamous cell carcinoma, which occurs after 15 to 25 years in the middle third of the thoracic esophagus in
10% of patients. The classic manometric findings of achalasia are failure of relaxation of the LES on swallowing and absent
or weak simultaneous contractions in the esophageal body after swallowing. Achalasia is currently incurable, and, though
the recently described endoscopic botulinum toxin injection of the lower esophageal sphincter, pneumatic dilatation, and
esophagomyotomy effectively relieve dysphagia in the majority of patients, all of these treatments are strictly palliative. The
motility disturbance persists throughout life.

7. Which of the following statements about diffuse esophageal spasm is/are true?
A. Chest pain due to esophageal spasm is readily differentiated from angina pectoris of cardiac origin.
B. Bouts of esophageal obstruction and regurgitation of food are characteristic.
C. Associated psychiatric disorders are common.
D. During manometric assessment, unless the patient is having pain there may be no detectable multiphasic, high-
amplitude, simultaneous esophageal contractions.
E. The treatment of choice is a long esophagomyotomy from the aortic arch to the esophagogastric junction.
Answer: CD

DISCUSSION: The chest pain of diffuse esophageal spasm is often indistinguishable from that of angina pectoris of cardiac
origin. Many patients undergo cardiac catheterization to rule out coronary artery disease. Patients may experience slow
emptying of the esophagus, but obstructive symptoms and regurgitation of food are unusual. Psychiatric disorders
(depression, psychosomatic complaints, anxiety) have been documented in more than 80% of patients with esophageal
contraction abnormalities. The classic manometric criteria of diffuse esophageal spasm are simultaneous, multiphasic,
repetitive, high-amplitude contractions occurring after a swallow and spontaneously. These changes may not be detected if
manometry is performed when the patient is having no pain. As the cause of esophageal spasm is unknown, treatment is far
from ideal. Conservative management—avoidance of “trigger” foods and drinks, psychiatric counseling, treatment of reflux,
esophageal dilatations, use of histamine H 2 blockers, anti-spasmodics, and smooth muscle relaxants—should always be
attempted first. Esophagomyotomy provides no lasting relief of esophageal spasm for as many as 50% of patients and
should be used only in a minority of these patients.

8. Which of the following statements about epiphrenic diverticula of the esophagus is/are correct?
A. They are traction diverticula that arise close to the tracheobronchial tree.
B. They characteristically arise proximal to an esophageal reflux stricture.
C. The degree of dysphagia correlates with the size of the pouch.
D. They are best approached surgically through a right thoracotomy.
E. The operation of choice is a stapled diverticulectomy, long esophagomyotomy, and partial fundoplication.
Answer: E

DISCUSSION: Epiphrenic diverticula are pulsion diverticula that arise in the distal 10 cm. of the esophagus. The cause is
elevated intraesophageal pressure, which forces mucosa and submucosa to herniate through the muscle layers. Though this
may occur with an esophageal reflux stricture or tumor, primary disordered esophageal motility is the most common cause.
Many patients are asymptomatic when their epiphrenic diverticula are diagnosed by barium esophagography, symptoms
being related more to the degree of disordered motility than to the size of the pouch. Distal esophageal diverticulum is best
approached through a left thoracotomy. When surgery is indicated, the preferred approach is transthoracic diverticulectomy
using a surgical stapler, a long esophagomyotomy to relieve the elevated intraesophageal pressure, and a nonobstructing
partial (e.g., Belsey's) fundoplication.

Asir Surgery MCQs Bank. © 1422H-2002- first impression ©


159
This project was raised after an idia by Dr. Gharama Al-Shehri (consultant surgeon). Developed and typed by Dr. Ghazi Al-Shumrani (intern).
Esophagus, stomach & duodenum
9. Which of the following statements about Schatzki's ring is correct?
A. The ring represents a panmural fibrotic stricture resulting from gastroesophageal reflux.
B. Dysphagia occurs when the ring diameter is 13 mm. or less.
C. The ring occurs within 1 to 2 cm. of the squamocolumnar epithelial junction.
D. Schatzki's ring indicates reflux esophagitis.
E. Schatzki's ring signifies the need for an antireflux operation.
Answer: B

DISCUSSION: Schatzki's ring is seen radiographically as an annular weblike constriction at the esophagogastric junction in
a patient with a sliding hiatalhernia. It represents prominence of the esophagogastric junction with slight submucosal
fibrosis but not true panmural fibrotic reflux stricture. Intermittent dysphagia may occur when the ring size is 20 mm. or
less, but dysphagia is almost invariable when the ring measures 13 mm. or less. Schatzki's ring occurs precisely at the
squamocolumnar epithelial junction. It is indicative of the presence of a hiatal hernia but not of gastroesophageal reflux or
esophagitis. An asymptomatic Schatzki's ring requires no therapy per se. Patients with refractory severe reflux symptoms
after dilation therapy are candidates for antireflux surgery.

10. Choose the distance in centimeters from the upper incisor teeth at which the following radiographically identified
esophageal lesions would be encountered endoscopically:
A. 10 cm. 1. Zenker's diverticulum
B. 15 cm. 2. Traction diverticulum
C. 25 cm. 3. Tumor 10 cm. proximal to the esophagogastric junction
D. 30 cm.
E. 40 cm.
Answer: 1-B, 2-C, 3-D

DISCUSSION: A barium esophagogram should be obtained routinely before performing elective esophagoscopy. The
location of an esophageal lesion seen on the barium swallow study can be related to adjacent anatomic landmarks. This
allows the endoscopist to anticipate the level (as measured from the upper incisor teeth) where he should expect to see the
abnormality at esophagoscopy. For example, the upper esophageal (cricopharyngeal) sphincter is generally seen at the level
of the C7–T1 vertebrae radiographically and at 15 cm. endoscopically. This is the level at which the mouth of a Zenker's
diverticulum is seen. The tracheal bifurcation occurs at the level of the T4 or 25 cm. from the upper incisors endoscopically;
a traction (parabronchial) diverticulum at the level of the carina on a barium esophagogram will be seen at approximately 25
cm. The esophagogastric junction occurs at approximately the level of T11, 40 cm. from the upper incisors; a tumor 10 cm.
proximal to the esophagogastric junction stricture is seen endoscopically at 30 cm.

11. Which of the following statements about pathology encountered at esophagoscopy is/are correct?
A. Reflux esophagitis should be graded as mild, moderate, or severe, to promote consistency among different observers.
B. An esophageal reflux stricture with a 2-mm. lumen is not dilatable and is best treated with resection.
C. A newly diagnosed radiographic distal esophageal stricture warrants dilation and antireflux medical therapy.
D. In patients with Barrett's mucosa, the squamocolumnar epithelial junction occurs 3 cm. or more proximal to the
anatomic esophagogastric junction.
E. After fasting at least 12 hours, a patient with megaesophagus of achalasia can safely undergo flexible fiberoptic
esophagoscopy.
Answer: D

DISCUSSION: The traditional subjective grading of reflux esophagitis as mild, moderate, or severe has inherent wide
variations in meaning among observers. Consistent use of standardized grading systems for endoscopic reflux esophagitis
(e.g., that of Belsey or Savary) provides a more objective description of the changes seen and allows more meaningful
evaluation of patients at different times and by different observers. The size of the lumen does not predict whether or not a
reflux stricture is dilatable. Even a tight 2-mm. lumen can be traversed with a guidewire over which Savary dilators can be
used to achieve an acceptable lumen size. Every newly diagnosed esophageal stricture warrants esophagoscopy with
brushings and biopsies of the stricture (to exclude carcinoma) and an assessment of its “dilatability.” Antireflux medical
therapy is not justified until carcinoma has been ruled out. Because the squamocolumnar epithelial junction may normally
be found within 2 to 3 cm. of the anatomic esophagogastric junction, the diagnosis of Barrett's mucosa requires
identification of the columnar epithelium at least 3 mm. proximal to the junction of the tubular esophagus and the stomach.
In advanced achalasia with megaesophagus, the dilated esophagus may have a capacity of 1 to 2 liters, and simply fasting
Asir Surgery MCQs Bank. © 1422H-2002- first impression ©
160
This project was raised after an idia by Dr. Gharama Al-Shehri (consultant surgeon). Developed and typed by Dr. Ghazi Al-Shumrani (intern).
Esophagus, stomach & duodenum
overnight does not ensure that the esophagus is empty of food and drink consumed the day before. Life-threatening massive
regurgitation and aspiration may occur as the endoscope is being introduced unless an effort is made to evacuate the
esophagus first by means of a nasogastric tube.

12. Which of the following statements about the diagnosis and treatment of esophageal leiomyomas is/are correct?
A. The majority are diagnosed after they cause dysphagia and chest pain.
B. Biopsy is indicated at the time of esophagoscopy, to rule out carcinoma.
C. Full-thickness elliptical excision of the esophageal wall is the preferred surgical approach.
D. Endoscopic ultrasonography is a reliable means of following leiomyomas conservatively.
E. Recurrence of resected leiomyomas is minimized by wide local excision.
Answer: D

DISCUSSION: Most esophageal leiomyomas are asymptomatic when discovered incidentally on a barium esophagogram or
upper gastrointestinal tract series. When suspected on the basis of its radiographic appearance, biopsy of the mass should
not be performed at the time of esophagoscopy, so that subsequent extramucosal resection will not be complicated by
scarring at the biopsy site. The preferred surgical approach is submucosal enucleation of the mass, not full-thickness
excision. Leiomyomas have a characteristic hypoechogenic homogeneous appearance on esophageal ultrasonography that
allows a noninvasive diagnosis and means of surveillance. Submucosal enucleation of leiomyomas, without wide local
excision, provides excellent long-term results with virtually no local recurrence rate.

13. Which of the following statements regarding the pathology of esophageal carcinoma is/are correct?
A. Worldwide, adenocarcinoma is the most common esophageal malignancy.
B. Squamous cell carcinoma is most common in the distal esophagus, whereas adenocarcinoma predominates in the middle
third.
C. Patients with Barrett's metaplasia are 40 times more likely than the general population to develop adenocarcinoma.
D. Metastases from esophageal carcinoma are characteristically localized to regional mediastinal lymph nodes adjacent to
the tumor.
E. Achalasia, radiation esophagitis, caustic esophageal stricture, Barrett's mucosa, and Plummer-Vinson syndrome are all
premalignant esophageal lesions that predispose to the development of squamous cell carcinoma.
Answer: C

DISCUSSION: Histologically, 95% of esophageal cancers worldwide are squamous cell carcinomas, but the incidence of
adenocarcinoma is increasing dramatically in the United States and Europe. Squamous cell carcinoma predominates in the
upper and middle thirds of the esophagus, whereas adenocarcinoma is the most frequent distal esophageal cancer. A
columnar lined lower esophagus (Barrett's metaplasia) is associated with an incidence of adenocarcinoma approximately 40
times greater than that of the general population. Esophageal cancer is a biologically aggressive tumor that characteristically
metastasizes widely to regional and distant lymph nodes as well as to liver and lungs. Recognized premalignant esophageal
lesions include achalasia, radiation esophagitis, caustic stricture, Plummer-Vinson syndrome, leukoplakia, esophageal
diverticula, and Barrett's metaplasia. All but Barrett's metaplasia are associated with the development of squamous cell
carcinoma.

14. Which of the following statements about the surgical treatment of esophageal carcinoma is/are correct?
A. The finding of severe dysphagia in association with Barrett's mucosa is an indication for an antireflux operation to
prevent subsequent development of carcinoma.
B. Long-term survival is improved by radical en bloc resection of the esophagus with its contained tumor, adjacent
mediastinal tissues, and regional lymph nodes.
C. The morbidity and mortality rates for cervical esophagogastric anastomotic leak are substantially less than those
associated with intrathoracic esophagogastric anastomotic leak.
D. The leading complications of transthoracic esophagectomy and intrathoracic esophagogastric anastomosis are bleeding
and wound infection.
E. Transhiatal esophagectomy without thoracotomy achieves better long-term survival than transthoracic esophagectomy.
Answer: C

DISCUSSION: Severe dysplasia in Barrett's mucosa is indicative of carcinoma in situ and is an indication for resectional
therapy, not an antireflux operation. In the majority of patients, local tumor invasion or distant metastases preclude cure
Asir Surgery MCQs Bank. © 1422H-2002- first impression ©
161
This project was raised after an idia by Dr. Gharama Al-Shehri (consultant surgeon). Developed and typed by Dr. Ghazi Al-Shumrani (intern).
Esophagus, stomach & duodenum
when esophageal carcinoma is diagnosed, and attempts to improve survival with a more radical local operation performed in
the face of systemic disease have been disappointingly futile. A cervical esophagogastric anastomotic leak causes a
relatively minor cervical salivary fistula that heals in 7 to 10 days in 95% of patients. In contrast, an intrathoracic
esophagogastric anastomotic leak results in mediastinitis, which is fatal in 50%. The leading complications of transthoracic
esophagectomy and an intrathoracic esophagogastric anastomosis are respiratory insufficiency (from combined thoracic and
abdominal incisions) and anastomotic leak resulting in mediastinitis and sepsis. Both complications are minimized by
transhiatal esophagectomy without thoracotomy plus cervical esophagogastric anastomosis. No single operative approach to
the treatment of esophageal cancer has proved superior to others in terms of long-term survival. The biologic behavior of the
tumor (its stage and aggressiveness)—not the number of lymph nodes resected with the tumor—determines survival.

15. The best management for a 48-hour-old distal esophageal perforation is:
A. Antibiotics and drainage.
B. Division of the esophagus and exclusion of the perforation.
C. Primary repair with buttressing.
D. Resection with cervical esophagostomy, gastrostomy, and jejunostomy.
E. T-tube fistula and drainage.
Answer: C

DISCUSSION: When the esophagus is repaired primarily and covered by well-vascularized autologous tissue, the rates of
fistula and death are significantly less than those observed for patients who receive simple repair without any protection.
Primary repair with buttressing is the first choice for treatment. Resection is reserved for esophageal perforations with
extensive damage to the esophageal wall or with advanced mediastinal infection and sepsis. Exclusion of the perforated
esophagus and T-tube drainage of a perforation are alternative approaches that cannot be considered for primary treatment.
Antibiotics and drainage as the sole treatment is reserved for a very small, selected population of patients with well-
contained esophageal perforation.

16. A 50-year-old patient develops sudden left lower chest pain and epigastric pain after vomiting. The patient shows
diaphoresis, breath sounds are decreased on the left, and there is abdominal guarding. The most appropriate diagnostic test
is:
A. Aortography.
B. Esophagoscopy.
C. Electrocardiogram.
D. Film of the chest.
E. White blood count.
Answer: D

DISCUSSION: The history of pain after vomiting efforts suggests esophageal rupture. Pain is often described as
excruciating and frequently masquerades as a dissecting aneurysm, perforated ulcer, or myocardial infarction. Decreased
breath sounds suggest the possibility of hydropneumothorax. The diagnostic procedure is a chest film. More than 90% of
patients with an esophageal perforation show abnormalities suggestive of perforation. The findings are influenced by the
interval between perforation and the examination, by the site of the perforation, and by the integrity of the mediastinal
pleura. Esophagoscopy is not indicated.

17. The following statements about the influence of diet and lifestyle on lower esophageal sphincter (LES) function are true
except one. Identify the incorrect statement.
A. A high-protein diet increases LES pressure.
B. A fat meal results in sustained decrease in LES pressure.
C. Chocolate ingestion causes a decrease in LES pressure.
D. Peppermint produces a transient decrease in LES values.
E. Cigarette smoking produces no significant changes in LES pressures.
Answer: E

DISCUSSION: There is a dramatic decrease in LES pressure following the ingestion of fat. Chocolate has the same effect
on LES resting pressures. Peppermint was shown to produce transient decreases in LES pressures of 20 to 30 seconds,
which occur approximately 10 minutes after ingestion. The ingestion of carbohydrates produces no significant change in
Asir Surgery MCQs Bank. © 1422H-2002- first impression ©
162
This project was raised after an idia by Dr. Gharama Al-Shehri (consultant surgeon). Developed and typed by Dr. Ghazi Al-Shumrani (intern).
Esophagus, stomach & duodenum
sphincter pressures, but a high-protein meal increases LES pressure. Cigarette smoking has also been shown to produce
significant decreases in LES pressure that persist throughout the duration of active smoking.

18. When a stricture is present in association with gastroesophageal reflux, each of the following is an acceptable repair for
reflux control except one. Identify the poorest repair.
A. Intrathoracic total fundoplication.
B. Lengthening gastroplasty with total fundoplication.
C. Total fundoplication.
D. Lengthening gastroplasty with partial fundoplication.
E. Partial fundoplication.
Answer: E

DISCUSSION: When a stricture is present, periesophagitis and shortening limit the chances of obtaining a sufficient length
of intra-abdominal esophagus. Even extensive mobilization of the esophagus to the aortic arch and freeing of the
esophagogastric junction does not afford a comfortable 4 to 5 cm. of esophagus under the diaphragm. A tension-free repair
is not possible in such circumstances. Partial fundoplication at this stage of the disease is followed by a 45% failure rate.
Excellent results have been reported using a total fundoplication following dilatation of the stricture, and intrathoracic
fundoplication provided good results. The lengthening gastroplasty with a partial fundoplication or with total fundoplication
shows satisfactory control of reflux in a majority of patients.

19. When assessing gastroesophageal reflux disease by manometry each of the following statements is correct except one.
Identify the incorrect one.
A. Absent or extremely low LES pressures have predictive value in identifying more severe reflux.
B. Peristaltic dysfunction increases with increasing severity of esophagitis.
C. With established reflux disease the UES is hypertensive.
D. Esophageal functional changes are worst in patients with a circumferential columnar-lined esophagus.
E. Absence of peristalsis may be associated with more severe forms of reflux disease.
Answer: C

DISCUSSION: In reflux disease when LES pressure is below 10 mm. Hg, manometry is too imprecise to identify a
potential for significant reflux. If the pressure is less than 6 mm. Hg, this shows a reasonable high specificity as compared
with abnormal reflux on pH testing. When LES pressure is extremely low or nonexistent, this identifies a more severe
degree of reflux and a poorer prognosis for long-term medical therapy. In the esophageal body, active reflux esophagitis
causes altered function. Failed peristalsis increases, and the contractions become weaker. Patients with a columnar-lined
esophagus have the worst functional abnormalities. Although distention or acid perfusion in the proximal esophagus can
produce a significant increase in UES resting pressure, there is at present no solid evidence relating UES resting pressures to
active reflux disease.

20. The presence of a nonmalignant mid- or upper esophageal stricture always indicates the presence of:
A. Alkaline reflux esophagitis.
B. Barrett's esophagus.
C. Idiopathic reflux disease.
D. Mediastinal fibrosis.
E. Scleroderma.
Answer: B

DISCUSSION: A stricture at or above the aortic arch is almost certainly situated above an esophagus lined at its lower end
with columnar epithelium. Barrett's esophagus is suggested radiographically when local esophagitis, ulcer, or stricture is at
the limits of a normal-looking segment of esophagus under the stricture but above a herniated stomach. The columnar-lined
esophagus is not always associated with a high stricture; however Messiaen and Halpert documented strictures in 80% of
their Barrett's patients. These high strictures, when seen with alkaline reflux esophagitis, with idiopathic reflux disease, or in
association with scleroderma, always suggest the presence of a columnar-lined esophagus. Mediastinal fibrosis is a rare
condition that can cause multilevel strictures on the esophagus.

Asir Surgery MCQs Bank. © 1422H-2002- first impression ©


163
This project was raised after an idia by Dr. Gharama Al-Shehri (consultant surgeon). Developed and typed by Dr. Ghazi Al-Shumrani (intern).
Esophagus, stomach & duodenum
21. Which of the following is most reliable for confirming the occurrence of a significant esophageal caustic injury?
A. History of the event.
B. Physical examination of the patient.
C. Barium esophagraphy.
D. Endoscopy.
Answer: D

DISCUSSION: In the absence of physical or radiographic evidence of upper airway obstruction or esophagogastric
perforation, the presence of a significant caustic injury can be defined reliably only by direct visualization at the time of
endoscopy. Although the history may shed light on the possibility of a burn and its severity, all too often the event goes
unwitnessed or the type and amount of ingested substance are not known with certainty. The identification of oropharyngeal
burns clearly indicates the need for endoscopy, but as many as 70% of patients with such lesions escape associated
esophageal injury. Conversely, 10% to 30% of patients with no external evidence of burns have subsequently been
confirmed by esophagoscopy to have sustained damage. In the absence of an identifiable perforation, a barium
esophagogram can rarely be considered unequivocally diagnostic of acute injury, though such an injury may be suggested
when the esophagus appears atonic and dilated, rigid and persistently narrowed, or excessively irritable. Because of the
importance of early confirmation of the presence or absence of a significant esophageal burn as a guide to formulating
appropriate treatment, esophagoscopy should be performed expeditiously as soon as sufficient time has elapsed to allow
gastric emptying and stabilization of the patient, preferably within the first 12 to 48 hours after ingestion.

22. Indications for surgical reconstruction of the esophagus include which of the following?
A. Continuing requirement for frequent dilation of an extensive esophageal stricture for a minimum of 2 years.
B. Failure or refusal of the patient to comply with a treatment regimen of regular dilation.
C. Development of a fistula between the esophagus and tracheobronchial tree.
D. Iatrogenic perforation of the esophagus during attempted dilation.
Answer: BCD

DISCUSSION: Development of a tracheobronchial fistula almost always necessitates some form of esophageal
reconstruction because of the extensive damage usually associated with it. Failure of the patient to cooperate effectively
condemns to failure any attempt at restoring esophageal patency by bougienage, because sporadic attempts at dilation do not
allow progressive lumen enlargement but, rather, invite additional injury because of the necessity for repeated
instrumentation of a recurrent, tight stricture. The need for repeated dilation of extensive or multiple strictures over a period
exceeding 6 months should prompt surgical reconstruction of the esophagus, especially in young children, for whom the
psychological and physical hazards are intensified by prolonged treatment. In such circumstances, a 2-year period of
attempted bougienage is excessive. Although iatrogenic perforation often signals the need for esophageal reconstruction,
this misadventure should not be considered an absolute indication but should be assessed in relation to (1) the extent and
complexity of the stricture, (2) the potential for eventually achieving successful bougienage, and (3) the severity of
complications caused by the secondary injury.

23. First-line therapy for routine peptic duodenal ulcer disease includes:
A. Vagotomy and antrectomy.
B. Upper endoscopy and biopsy to rule out tumor.
C. Evaluation for Helicobacter pylori.
D. Serum gastrin determination.
E. Cream or milk-based “Sippy” diet.
Answer: C

DISCUSSION: Vagotomy and antrectomy is the definitive surgical therapy for peptic ulcer disease but should be applied
only for complications of the disease or after refractory disease has been documented. Biopsy of routine peptic duodenal
ulcer is not indicated to rule out malignancy except in special circumstances, such as an endoscopic appearance typical of
malignancy. H. pylori is found in a large percentage of peptic ulcer patients, and treatment alters the rate of recurrence if
therapy is directed toward reduction of H. pylori in addition to acid. Measurement of serum gastrin is recommended for
patients with resistant or persistent peptic ulcer disease for patients undergoing surgery for peptic ulcer disease. The Sippy
diet has not been recommended clinically for years. Formerly it was recommended as a bland diet that would not exacerbate
peptic ulcer disease. It is now known that these diets are heavily calcium-laden and probably exacerbate peptic ulcer
disease.
Asir Surgery MCQs Bank. © 1422H-2002- first impression ©
164
This project was raised after an idia by Dr. Gharama Al-Shehri (consultant surgeon). Developed and typed by Dr. Ghazi Al-Shumrani (intern).
Esophagus, stomach & duodenum

24. Appropriate management of severe vomiting associated with gastric outlet obstruction from peptic ulcer disease
includes all of the following except:
A. Nasogastric suction.
B. Intravenous hydration.
C. Nutritional assessment; upper endoscopy to rule out malignancy.
D. Intravenous H 2 antagonist.
E. Oral antacid therapy.
Answer: E

DISCUSSION: All patients should undergo nasogastric suction, rehydration, and control of acid secretion. This control of
acid secretion requires an H 2 antagonist since oral antacids are often inadequate to neutralize the large volume of acid often
present in the obstructed stomach. Patients with a long history of obstruction are often nutritionally compromised and need
careful nutritional assessment before operative planning.

25. All of the following are complications of peptic ulcer surgery except:
A. Duodenal stump blowout.
B. Dumping.
C. Diarrhea.
D. Delayed gastric emptying.
E. Steatorrhea.
Answer: E

DISCUSSION: Duodenal stump blowout occurs after Billroth II operations, where back-pressure in the duodenal stump
results in breakdown of this stump closure, leading to abdominal sepsis. Dumping syndrome and postvagotomy diarrhea are
complications of peptic ulcer surgery. They represent two different syndromes, both of which are predominately related to
the vagotomy portion of the operation. Delayed gastric emptying occurs frequently after peptic ulcer surgery, for a variety of
reasons, but it is most common after elective peptic ulcer surgery for gastric outlet obstruction. Steatorrhea is not
necessarily related to peptic ulcer surgery but is a complication of pancreatic insufficiency.

26. The presentation of Zollinger-Ellison syndrome includes all of the following except:
A. Hyperparathyroidism in patients with multiple endocrine neoplasia type 1 (MEN 1) syndrome.
B. Diarrhea.
C. Migratory rash.
D. Jejunal ulcers.
E. Duodenal ulcers.
Answer: C

DISCUSSION: Zollinger-Ellison syndrome occurs in two settings: sporadically and in association with MEN 1 syndrome.
MEN 1 syndrome includes parathyroid adenomas, and the initial presentation is often related to this parathyroid disease.
Diarrhea is a common presentation for Zollinger-Ellison syndrome, since hyperacidity can result in diarrhea due to the
volume of acid secreted or from a steatorrhea-type diarrhea when the high levels of acids inactivate the pancreatic enzymes.
Migratory rash is commonly associated with glucagonoma but not with Zollinger-Ellison syndrome. Both jejunal and
duodenal ulcers can be found with Zollinger-Ellison syndrome.

27. All are true about the dumping syndrome except:


A. Symptoms can be controlled with a somatostatin analog.
B. Diarrhea is always part of the dumping syndrome.
C. Flushing and tachycardia are common features of the syndrome.
D. Separating solids and liquids in the patient's oral intake alleviates some of the symptoms of the syndrome.
E. Early postoperative dumping after vagotomy often resolves spontaneously.
Answer: B

Asir Surgery MCQs Bank. © 1422H-2002- first impression ©


165
This project was raised after an idia by Dr. Gharama Al-Shehri (consultant surgeon). Developed and typed by Dr. Ghazi Al-Shumrani (intern).
Esophagus, stomach & duodenum
DISCUSSION: The somatostatin analog octreotide has been used to control the dumping syndrome and is currently the only
known medical therapy for this disease. Other therapies include dietary measures such as six small meals a day and
separation of solids and liquids. Postvagotomy diarrhea is a secondary complication of vagotomy and is not strictly
associated with the dumping syndrome itself. The vast majority of patients with dumping syndrome experience spontaneous
resolution of their symptoms without intervention in the postoperative period.

28. In patients with bleeding duodenal ulcers, the endoscopic finding associated with the highest incidence of
rebleeding is:
A. Visible vessel.
B. Cherry-red spot.
C. Clean ulcer bed.
D. Duodenitis.
E. Shallow, 3-mm. ulcer.
Answer: A

DISCUSSION: A visible vessel in an ulcer bed is associated with a 50% chance of rebleeding and, other than an actively
bleeding vessel, is the worst endoscopic prognostic indicator for rebleeding. Cherry-red spot, adherent clot, and clean small
ulcers all are associated with a lower incidence of rebleeding.

29. All of the following are contraindications for highly selective vagotomy except:
A. Intractable duodenal ulcer disease.
B. Peptic ulcer disease causing gastric outlet obstruction.
C. Fundic peptic ulceration.
D. Cigarette chain smoking.
E. Perforated peptic ulcer disease with more than 24 hours' soilage.
Answer: A

DISCUSSION: Intractable peptic ulcer symptoms are a classic indication for highly selective vagotomy. Patients with
gastric outlet obstruction often do poorly with highly selective vagotomy and develop recurrent ulceration. Highly selective
vagotomy is not indicated for gastric ulceration. Heavy chain smokers often get recurrent peptic ulceration after highly
selective vagotomy; therefore, vagotomy and antrectomy is indicated for them. Patients who experience long periods of
perforation before exploratory laparotomy should receive either patch plus vagotomy or pyloroplasty or patch of the ulcer
alone. Extensive operations, such as highly selective vagotomy, are usually not indicated in this acute setting.

30. All the following are true of omeprazole except:


A. It is the only drug available that has the potential to achieve pharmacologically induced achlorhydria.
B. It works by blocking the hydrogen-potassium ATPase in the parietal cell.
C. It is parietal cell specific.
D. It has a short half-life (about 90 minutes) when taken orally.
E. It has been associated with gastric neoplasm in a rat model.
Answer: D

DISCUSSION: Omeprazole and drugs in this category are the only drugs that can produce achlorhydria. All other antiacid
drugs reduce acid secretion without producing achlorhydria. Omeprazole inhibits acid at the final common pathway by
blocking the hydrogen-potassium ATPase in parietal cells. It is gastric parietal cell specific and has a very long half-life,
allowing once daily dosing when given orally. When it was given to rats in pharmacologic doses the gastric mucosa formed
carcinoid-type tumors. This problem has not been identified in humans.

31. All of the following statements about gastrin-releasing peptide (GRP) are true except:
A. In species other than man and dog GRP is commonly referred to as bombesin.
B. GRP serves as a neurotransmitter.
C. GRP inhibits pancreatic secretion when given intravenously.
D. GRP stimulates gastric acid secretion when given intravenously.
E. GRP is released in response to cholinergic stimulation of the parietal cells to stimulate release of gastrin.
Asir Surgery MCQs Bank. © 1422H-2002- first impression ©
166
This project was raised after an idia by Dr. Gharama Al-Shehri (consultant surgeon). Developed and typed by Dr. Ghazi Al-Shumrani (intern).
Esophagus, stomach & duodenum
Answer: C

DISCUSSION: Gastrin-releasing peptide and bombesin are homologous peptides of different amino acid lengths. GRP
functions as a neurotransmitter at the cholinergic nerve ending on the parietal cell and releases gastrin after cholinergic
stimulation. It functions to increase gastric acid secretion and also pharmacologically increases pancreatic secretion.

32. Cholecystokinin (CCK) is believed to function in all of the following processes except:
A. It physiologically delays gastric emptying.
B. It appears to have a role in satiety regulation.
C. It contracts the gallbladder.
D. It stimulates pancreatic secretion.
E. It is important in the control of the anal sphincter.
Answer: E

DISCUSSION: CCK has a physiologic role in the regulation of gastric emptying, eating behavior, gallbladder contraction,
and pancreatic secretion. There is experimental evidence that it may serve as a neurotransmitter in the function of the lower
esophageal sphincter. It probably also has a role in augmenting the release of insulin after a meal. It has no known role in
the function of the anal sphincter.

33. All of the following measures have been recommended for control of acid secretion in patients with Zollinger-
Ellison syndrome except:
A. Antrectomy.
B. Highly selective vagotomy.
C. Total gastrectomy.
D. Vagotomy and pyloroplasty.
E. Medical therapy with Prilosec (omeprazole).
Answer: A

DISCUSSION: Patients with MEN 1 syndrome or sporadic-metastatic Zollinger-Ellison syndrome should be palliated with
omeprazole to control their acid secretion. Patients who undergo exploration may have a variety of operations to control
their ulcer diathesis, including total gastrectomy or various vagotomy-type operations. Antrectomy alone is not indicated,
since the gastrin that is contributing to the production of acid is not coming from the antrum but coming from the tumor.

34. All of the following contribute to peptic ulcer disease except:


A. Cigarette smoking.
B. Nonsteroidal anti-inflammatory drugs.
C. Helicobacter pylori.
D. Gastrinoma.
E. Spicy foods.
Answer: E

DISCUSSION: Cigarettes and nonsteroidal anti-inflammatory drugs are common contributors to peptic ulceration. H. pylori
is found in most patients with peptic ulceration, and eradication of this bacterium decreases the recurrence rate for peptic
ulcer disease. Gastrinoma results in much acid secretion and commonly presents with peptic ulcer disease. Dietary factors
such as spicy foods have little or no effect on postprandial acid secretion and do not contribute to peptic ulceration.

35. Which of the following statements about gastric polyps is/are true?
A. Like their colonic counterparts, gastric epithelial polyps are common tumors.
B. They are analogous to colorectal polyps in natural history.
C. Endoscopy can uniformly predict the histology of a polyp based on location and appearance.
D. In a given patient, multiple polyps are generally of a single histologic type.
E. Gastric adenomatous polyps greater than 2 cm. in diameter should be excised because of the risk of malignant
transformation.
Answer: DE
Asir Surgery MCQs Bank. © 1422H-2002- first impression ©
167
This project was raised after an idia by Dr. Gharama Al-Shehri (consultant surgeon). Developed and typed by Dr. Ghazi Al-Shumrani (intern).
Esophagus, stomach & duodenum

DISCUSSION: As early as 1895 Hauser reported an association between familial adenomatous polyposis of the colon and
multiple gastric polyps. This early association may have given rise not only to the confusing nomenclature of gastric polyps
but also to the mistaken notion that they are analogous to colorectal polyps in microscopic appearance and natural history.
Unlike colonic polyps, gastric epithelial polyps are very uncommon tumors (prevalence 0.4% to 0.8%). Their histologic
appearance cannot be predicted on the basis of location in the stomach, although the endoscopic literature is beginning to
define predictive algorithms based on location and ultrasound. Multiple polyps are almost always of a single histologic type.
Gastric adenomatous polyps have long been associated with adenocarcinoma. This association is directly related to the size
of the polyps. Up to 24% of polyps 2 cm. or greater in diameter are associated with adenocarcinoma. In contrast, only 4% of
polyps with a diameter less than 2 cm. are associated with carcinoma. The risk, if any, of carcinoma in patients with
hyperplastic polyps appears to be associated with the atrophic gastritis that frequently accompanies them rather than with
the polyps themselves.

36. Which of the following statements about gastric leiomyomas is/are true?
A. They are the most common type of gastric tumor of the stomach at autopsy.
B. The leiomyoblastoma cell type reflects malignant transformation of gastric leiomyomas.
C. A conservative surgical approach is indicated for their resection since regional lymphadenectomy has not been proved
reliable even when they turn out to be malignant.
D. Severe hemorrhage may occur from deep ulcerations overlying the intramural tumor.
Answer: ACD

DISCUSSION: Approximately 40% of benign tumors of the stomach are leiomyomas derived from the smooth muscle of
the stomach or its associated blood vessels. Because it is rare for gastric leiomyomas smaller than 3 cm. in diameter to be
symptomatic, considerably fewer than 2% of gastric neoplasms resected surgically are of smooth muscle origin. Gastric
leiomyomas may be smooth or lobulated, but in time a central ulceration occurs in the mucosal bulge of the tumor in
approximately half of submucosal leiomyomas. Ulceration may be present in smaller tumors but absent in very large
tumors. Overlying central mucosal ulceration, which may penetrate deeply into the tumor, results in hematemesis, melena,
or anemia and draws attention to the tumor. Bleeding from the tumor may be massive and/or intermittent.
Gastric leiomyomas are not encapsulated, even though on section they appear to be well-circumscribed. Microscopically,
the tumor cells at the margin may intermingle with cells of the surrounding gastric wall. Along with the presence of
occasional large cells with hyperchromatic nuclei, this has led to confusion in distinguishing benign tumors from malignant
ones. Stout described a reasonably distinct variety of gastric smooth muscle tumor that he called leiomyoblastoma (bizarre
smooth muscle tumor). They were characterized histologically by polyhedral smooth muscle cells with central nuclei and
abundant cytoplasm rather than elongated cells. A clear zone that surrounds the central nucleus may be an artifact of
fixation. Leiomyoblastoma may be benign or malignant. Carney has described a syndrome characterized by the triad of
multiple malignant leiomyoblastoma, pulmonary chondroma, and functioning extra-adrenal paraganglioma.
The principle of surgical treatment of smooth muscle tumors is local excision with a 2- to 3-cm. margin of surrounding
gastric wall. In view of the difficulty in distinguishing between the benign and malignant variants, enucleation is not an
appropriate method of treatment. Regional lymphadenectomy is not of proven value, even if malignancy is strongly
suspected and is not consistent with the known property of these tumors to spread by the hematogenous route.

37. The sine qua non of the histologic diagnosis of a gastric pseudolymphoma is:
A. Extragastric extension of the gastric lesion.
B. Nodal involvement beyond the immediate stomach.
C. A germinal center in the gastric lesion.
D. Extension into esophagus and duodenum.
E. Unresponsive to conservative gastric resection.
Answer: C

DISCUSSION: Pseudolymphoma represents approximately 10% of all gastric lymphomas. These are benign conditions
involving mostly the mucosa without evidence of nodal disease and without extragastric extension. The sine qua non for the
diagnosis is a germinal center within the gastric lesion. These are premalignant lesions but can be cured completely with
conservative resection.

38. All of the following statements about surgical management of gastric lymphomas are true except:
Asir Surgery MCQs Bank. © 1422H-2002- first impression ©
168
This project was raised after an idia by Dr. Gharama Al-Shehri (consultant surgeon). Developed and typed by Dr. Ghazi Al-Shumrani (intern).
Esophagus, stomach & duodenum
A. Stage I gastric lymphomas (small lesions confined to the stomach wall) can be cured completely with surgical therapy
alone.
B. Extensive gastric lymphomas that initially are treated with radiation and/or chemotherapy occasionally perforate during
treatment and require secondary resection.
C. Patients explored with a presumptive diagnosis of gastric lymphoma should undergo an attempt at curative resection
when this is safe and feasible.
D. Without a preoperative diagnosis resection for gastric mass should not be attempted unless lymphoma can be excluded.
E. Appropriate staging for primary gastric lymphoma includes bone marrow biopsy.
Answer: D

DISCUSSION: Operation alone is adequate treatment for very early-stage lymphoma, although chemotherapy is commonly
added. For more advanced disease, particularly stages III and IV, preoperative radiation chemotherapy is often indicated,
even though some of these patients suffer perforation during therapy and require emergent resection. Patients who undergo
exploration for gastric mass without a preoperative diagnosis can safely be resected with potential for cure even if the
diagnosis includes gastric lymphoma.

39. Which of the following risk factors have been shown to increase significantly the incidence of gastrointestinal
bleeding from stress gastritis in intensive care unit (ICU) patients?
A. Glucocorticoid administration.
B. Respiratory failure.
C. Coagulopathy.
D. Organ transplantation.
E. Jaundice.
Answer: BC

DISCUSSION: Prophylactic measures such as H 2-receptor antagonists and antacid titration effectively reduce the
incidence of gastrointestinal bleeding; however, prophylaxis against stress gastritis is expensive and may have adverse
effects. Therefore, it should be used selectively in patients with high risk factors. In a prospective multicenter study in which
10 potential risk factors were evaluated for stress gastritis bleeding in ICU patients, respiratory failure and coagulopathy are
two independent risk factors for clinically significant bleeding. Therefore, a strong recommendation for prophylaxis of
stress gastritis can be made for ICU patients who have either respiratory failure or coagulopathy.

40. Which of the following measures are effective in preventing stress gastritis bleeding in critically ill patients?
A. Improving systemic circulation by correcting any shocklike state resulting from blood loss or sepsis.
B. Correcting systemic acid-base abnormality.
C. Maintaining adequate nutrition.
D. Reducing intragastric acidity by either antacid titration or H 2 antagonists.
Answer: ABCD

DISCUSSION: Despite the lack of documentation, a strong impression exists among clinicians and clinical investigators
that the incidence and prevalence of stress gastritis have decreased significantly during the past decade, perhaps owing to
improved general care for critically ill patients. The improvement in general care of these critically ill patients includes
vigorous efforts to correct any shocklike state secondary to blood loss or sepsis, better ventilatory support, and maintenance
of adequate nutrition. These prophylactic measures enhance the ability of the gastric mucosa to protect itself against acid
injury. In addition, several prospective, randomized studies have shown that antacid titration and/or H 2-receptor antagonists
are effective in preventing gastrointestinal bleeding in these patients.

41. Which of the following have been used successfully to treat patients with vascular compression of the duodenum?
A. Subtotal gastrectomy and Roux-en-Y gastrojejunostomy.
B. Total parenteral nutrition.
C. Division of the ligament of Treitz and duodenal mobilization.
D. Percutaneous endoscopic gastrostomy.
E. Duodenojejunostomy.
Answer: BCE

Asir Surgery MCQs Bank. © 1422H-2002- first impression ©


169
This project was raised after an idia by Dr. Gharama Al-Shehri (consultant surgeon). Developed and typed by Dr. Ghazi Al-Shumrani (intern).
Esophagus, stomach & duodenum
DISCUSSION: Vascular compression of the duodenum is best treated initially with supportive care. Of paramount
importance is supplying adequate nutrition, since most patients have significant weight loss with this syndrome. This can
best be done with a nasojejunal feeding tube placed past the ligament of Treitz (and the obstructed area). Gastrostomy alone
does not provide unobstructed enteral access. Parenteral nutrition may be used successfully when enteral access cannot be
established. When operative therapy is needed, duodenojejunostomy has been the most common and successful operation
and is the treatment of choice for adults. In the pediatric population, division of the ligament of Treitz and duodenal
mobilization has also proved successful. Gastrojejunostomy has been used, but with a lower overall success rate. Distal
gastrectomy usually worsens duodenal obstruction by preventing duodenogastric reflux.

42. Which of the following statements about the anatomic basis for the syndrome of vascular compression of the
duodenum are true?
A. The duodenum is obstructed in its distal third as it crosses over the lumbar vertebral column.
B. Structures crossing beneath the superior mesenteric artery include the duodenum, the uncinate process of the pancreas,
and the left renal vein.
C. Hyperextension of the back allows the angle of origin of the superior mesenteric artery to widen, lessening the
obstruction of the duodenum.
D. Patients are at significant risk for vascular compression of the duodenum if the angle between the takeoff of the superior
mesenteric artery and the aorta is less than 45 degrees.
E. Arteriographic studies show a typical area of extrinsic compression and narrowing of the arterial lumen due to duodenal
pressure.
Answer: AB

DISCUSSION: The superior mesenteric artery originates behind the neck of the pancreas at the level of the first lumbar
vertebra. It arises from the aorta at an acute angle, usually about 37 degrees in normal patients, through which passes the left
renal vein, the uncinate process of the pancreas, and the distal third of the duodenum. The duodenum crosses the lumbar
spine from right to left and passes upward. It is at this point of passage of the duodenum upward and over the spine that the
obstruction occurs. Arteriographic studies show that the aortomesenteric angle in patients with the syndrome is only about 8
degrees. There is no narrowing of the superior mesenteric artery or disturbance of arterial flow, but the area of duodenal
obstruction corresponds to the compression of the bowel by the artery. The duodenal compression may often be relieved by
assuming the knee-chest, the left lateral, or even the prone position. Increasing lumbar lordosis, as with hyperextension of
the back, exacerbates the problem.

43. Numerous epidemiologic associations have been made between (1) environmental and dietary factors and (2) the
incidence of gastric cancer, including all except:
A. Dietary nitrites.
B. Dietary salt.
C. Helicobacter pylori infection.
D. Dietary ascorbic acid.
Answer: D

DISCUSSION: Numerous epidemiologic studies support the role of certain foods in the development of gastric cancer. Salt,
which can act as a gastric irritant, and nitrates and nitrites, which can be converted to the active carcinogens N-nitrosamines,
are implicated in the development of gastric cancer. H. pylori infection is associated with atrophic gastritis, a known
precursor to gastric cancer. Important studies of large populations indicate that the majority of patients with gastric cancer
are H. pylori positive. The bacteria produce toxins such as ammonia and acetaldehyde, which could lead to chronic
inflammation and epithelial damage. Dietary ascorbic acid has been associated with overall improvements in diet and is not
associated with the development of gastric cancer.

44. All of the following benign conditions are associated with increased rates of gastric cancer except:
A. Pernicious anemia.
B. Multiple endocrine neoplasia type I (MEN 1).
C. Adenomatous polyps.
D. Chronic atrophic gastritis.
Answer: B

Asir Surgery MCQs Bank. © 1422H-2002- first impression ©


170
This project was raised after an idia by Dr. Gharama Al-Shehri (consultant surgeon). Developed and typed by Dr. Ghazi Al-Shumrani (intern).
Esophagus, stomach & duodenum
DISCUSSION: Adenomatous polyps are unusual but carry the distinct potential for malignancy. They occur most often
between the fifth and seventh decades of life. The adenocarcinoma sequence in gastric polyps is thought to be analogous to
that of colonic polyps. An adenomatous polyp is a marker for increased risk of carcinoma in the remaining stomach. Both
pernicious anemia and chronic atrophic gastritis are associated with gastric cancer. Many of these patients develop chronic
achlorhydria, a condition also associated with an increased risk of cancer. Neither multiple MEN 1 nor MEN 2, is associated
with gastric cancer.

45. Which of the following statements concerning the pathology of gastric cancer is true?
A. Distal gastric cancers are becoming more common.
B. Intestinal-type gastric tumors resemble colon carcinomas and have a better prognosis than diffuse type.
C. Early gastric cancers are confined to the mucosa and are lymph node negative.
D. Broders' histologic grading system correlates well with survival: patients with grade IV tumors have 5-year survival
rates around 65%.
Answer: B

DISCUSSION: Distal gastric cancers are decreasing in incidence in several populations. Lesions of the gastroesophageal
junction and cardia have increased in incidence over the past two decades. Early gastric cancers are confined to the mucosa
and submucosa of the stomach. Six to 10% of these early lesions are lymph-node positive. The survival rates from early
gastric cancer is related to node positivity, just as in advanced gastric cancer. Broders' histologic grades do correlate well
with survival. Grade I and IV tumors are associated with a 65% and 11% 5-year survival, respectively. The Lauren
classification system is divided into intestinal and diffuse-type tumors. The intestinal-type tumor is more analogous to colon
carcinoma and has a better prognosis than the diffuse type.

46. An 80% distal gastrectomy is performed for a 6-cm. antral cancer with extension to the muscularis propria and
three positive lymph nodes less than 3 cm. from the tumor. The stage of this tumor was:
A. Stage I.
B. Stage II.
C. Stage III A.
D. Stage III B.
Answer: B

DISCUSSION: The American Joint Committee on Cancer Staging system depends on primary tumor, lymph node
involvement, and distant metastasis. The tumor described is a T2N1M0 tumor, which categorizes it as stage II.

47. Which of the following statements about the surgical treatment of gastric cancer is false?
A. Patients with tumors of the middle and proximal thirds should undergo total gastrectomy.
B. Adenocarcinoma of the cardia-gastroesophageal junction may require reconstruction in the abdomen, chest, or neck.
C. Palliative resection yields better results than palliative bypass.
D. Japanese patients who undergo gastric resection are, on average, 10 years younger and much leaner than their Western
counterparts.
Answer: A

DISCUSSION: Depending on the size and extent of the tumor, cancers of the gastroesophageal junction may extend
proximally into the esophagus for a varying distance. Reconstruction may be required in the abdomen, chest, or neck,
depending on extension and whether the operation is to be palliative or curative. Palliative resection yields better results
than palliative bypass, which is unreliable for relieving obstruction. Japanese patients typically are younger and thinner than
their Western counterparts. In addition, they have a higher prevalence (up to 50%) of early gastric cancer. Depending on the
size and particular location of the tumor, patients with small middle-third tumors or small lesions of the cardia may undergo
subtotal proximal gastrectomy and reconstruction with a gastric tube. If 6-cm. margins can be obtained on either side of the
lesion, total gastrectomy is unnecessary and may be associated with a higher risk of morbidity.

48. Which of the following measures of obesity correlates best with mortality?
A. The 1983 Metropolitan Life Insurance Company tables for ideal body weight.
B. Hydroimmersion measurements of body fat composition.
Asir Surgery MCQs Bank. © 1422H-2002- first impression ©
171
This project was raised after an idia by Dr. Gharama Al-Shehri (consultant surgeon). Developed and typed by Dr. Ghazi Al-Shumrani (intern).
Esophagus, stomach & duodenum
C. Body mass index (BMI).
D. Skinfold thickness.
E. Waist to hip ratios (WHR).
Answer: C

DISCUSSION: The measurement of obesity is still an inexact science. The Metropolitan Life Insurance tables, although
widely used, do not distinguish between lean muscle mass and fat. Accordingly, the BMI (weight in kg./height in meters) 2,
was developed to place greater emphasis on “fatness.” The measure correlates linearly with mortality tables.
Hydroimmersion data are still too sparse to relate to outcome tables. Skinfold thickness and waist-hip ratios have not been
shown to have the accuracy or relevance of the BMI.

49. The most effective therapy for morbid obesity, in terms of weight control, is:
A. Intensive dieting with behavior modification.
B. A multidrug protocol with fenfluramine, phenylpropanolamine, and mazindol.
C. A gastric bypass with a 40-ml. pouch, a 10- to 20-cm. Roux-en-Y gastroenterostomy.
D. A gastric bypass with a 15-ml. pouch, a 40- to 60-cm. Roux-en-Y gastroenterostomy.
E. Daily exercise with strong emphasis on utilizing all four limbs.
Answer: D

DISCUSSION: Although the various nonsurgical measures listed in the question have proved effective for obese persons,
they work only rarely for those who are morbidly obese. None have proved as effective as gastric bypass with a 15-ml.
pouch and a 40- to 60-cm. Roux-en-Y gastroenterostomy.

50. Which of the following statements about intestinal bypass is/are correct?
A. The operation produced weight loss similar to that of the gastric bypass.
B. The operation produced severe metabolic disturbances, including hypocalcemia, increased bile salts and glycine
synthesis.
C. Bacterial overgrowth in the bypassed segment led to liver failure.
D. The operation demonstrated that an adult human could survive with 40 to 50 cm. of small intestine.
Answer: ABCD

DISCUSSION: Unfortunately, all of the answers are true. Even though the intestinal bypass proved initially to be an
effective procedure to induce weight loss, the side effects proved to be so severe that almost all of the operations had to be
reversed or revised to gastric bypass to avert death from liver failure or severe illnesses due to malnutrition.

51. Which of the following is/are contraindications to gastric bypass surgery?


A. Diabetes mellitus.
B. Hypertension.
C. Pickwickian syndrome.
D. Failure to agree to long-term follow-up.
E. Sleep apnea.
Answer: D

DISCUSSION: The gastric bypass represents the best known therapy for diabetes mellitus, hypertension, the Pickwickian
syndrome, and sleep apnea. In fact, no other therapy provides such complete control of hyperglycemia and
hyperinsulinemia, reversal of hypertension, and total correction of the Pickwickian syndrome and most cases of sleep apnea.
The only contraindication to bariatric surgery listed is failure to agree to long-term follow-up. The gastric bypass represents
controlled malnutrition and, therefore, vitamin therapy is especially important. If patients are not followed closely, vitamin
deficiencies, especially of B 6 and B 12, can develop with serious consequences including a Korsakoff-Wernicke syndrome.

52. A 34-year-old morbidly obese diabetic woman underwent a gastric bypass about 12 hours ago. The operation was
technically difficult but finally went well. You are called because she now has a temperature of 99.2‫ ؛‬F, pulse of
134, and some pain in her incision and her back. She looks well; the incision is clean; and her examination is

Asir Surgery MCQs Bank. © 1422H-2002- first impression ©


172
This project was raised after an idia by Dr. Gharama Al-Shehri (consultant surgeon). Developed and typed by Dr. Ghazi Al-Shumrani (intern).
Esophagus, stomach & duodenum
otherwise negative. A bolus of 500 ml. of dextrose/lactated Ringer's did not change her vital signs, except that her
pulse rose to 140 without an increase in urine output. Your next step should be:
A. Another bolus of crystalloids.
B. Posterioanterior and lateral chest films.
C. Obtain white cell count, differential count, and electrolyte values.
D. Call the operating room and warn them that you need to re-explore for a leak.
E. Increase her pain medication.
Answer: D

DISCUSSION: Morbidly obese patients are malnourished and brittle and have little resistance. Serious life-threatening
infections may soil the peritoneal cavity without producing any sign except a persistently high pulse rate. The usual tests
listed in A, B, and C, should be done, but the most likely explanation for a continued high pulse is soiling in the area of the
surgery due to either a leak or contamination and development of sepsis. Because of the unreliability of clinical evaluation,
the indications for re-exploration are very liberal, and this approach has saved a number of lives. The risk of such an
exploration is small, whereas failure to contain the infection with lavage and drainage may be followed by a surprisingly
rapid death.

53. Metabolic complications of subtotal gastrectomy with Billroth I or Billroth II reconstruction include:
A. Hypothyroidism.
B. Anemia.
C. Reactive hypoglycemia.
D. Dumping syndrome.
E. Metabolic bone disease.
Answer: BCDE

DISCUSSION: Anemia develops in as many as 30% of patients within 15 years of surgery. The cause is multifactorial and
includes malabsorption of iron, folate, and vitamin B 12. A metabolic bone disease occurs in as many as 33% of patients, is
similar to osteomalacia, and is probably a result of malabsorption of calcium and vitamin D. Reactive hypoglycemia occurs
with rapid gastric emptying, resulting in increased glucose absorption immediately after a meal. Initially there is
hyperglycemia, leading to hyperinsulinemia and subsequent rapid glucose clearance and symptomatic hypoglycemia.
Dumping syndrome varies from very mild symptoms to significantly disabling ones. The severe syndrome occurs in fewer
than 5% of patients. Small, frequent, dry meals of low osmolality reduces symptoms, and somatostatin analog has been of
some clinical use.

54. Which of the following statement(s) concerning the surgical options for an anti-reflux operation is/are true?

a.A patient with normal esophageal length and esophageal body motility is best served by laparoscopic Nissen
fundoplication
b.A patient with a low peristaltic amplitude of the distal third of the esophagus is a candidate for an open Nissen
fundoplication
c.A Collis gastroplasty is an additional procedure that can be added in patients with extensive esophageal shortening
d.End-stage reflux disease such as an undilatable stricture or Barrett’s esophagus with high grade dysplagia is best
managed by a colon interposition
Answer: a, c, d

Patients with normal esophageal length and normal esophageal body motility are best served by a transabdominal Nissen
fundoplication. This is now normally done via the laparoscopic route. If the patient is very obese or requires concomitant
surgery on the lung or esophageal body, the transthoracic approach is preferable. The presence of a motility disorder alters
the operative strategy. If the peristaltic amplitude is low (20 mm Hg) in the distal third of the esophagus, a Nissen
fundoplication would create too much resistance and lead to dysphagia. In this situation the Belsey fundoplication is a better
choice. Moreover, it allows the surgeon to mobilize the esophagus to a much greater extent than is possible through the
abdomen. In addition to extensive mobilization, a Collis gastroplasty can be created to produce an extra 5 cm of “neo-
esophagus” around which a Belsey procedure can be added. End-stage reflux disease, for example, when there is an
undilatable stricture or after previous unsuccessful anti-reflux operations or when Barrett’s esophagus leads to high grade

Asir Surgery MCQs Bank. © 1422H-2002- first impression ©


173
This project was raised after an idia by Dr. Gharama Al-Shehri (consultant surgeon). Developed and typed by Dr. Ghazi Al-Shumrani (intern).
Esophagus, stomach & duodenum
dysplagia, is best served by esophageal replacement. The most durable substitute is the colon, and the functional results are
especially good if the vagus nerves are intact.

55. Factors associated with the development of complications of gastroesophageal reflux disease include:

a.The presence of a defective lower esophageal sphincter


b.Inadequate esophageal clearance
c.The presence of a hiatal hernia
d.The presence of an alkaline component of the reflux material
Answer: a, b, c, d

The status of the lower esophageal sphincter (LES) has emerged as a significant factor in several long-term studies of
gastroesophageal reflux disease, and serves as a predictor of a poor response to medical treatment. Barrett’s esophagus is
almost always associated with a mechanically defective sphincter. Any defects in esophageal clearance which prolongs the
contact time between the refluxate and the mucosa is likely to lead to increased esophageal injury. The presence of a hiatal
hernia is also associated with more complications of gastroesophageal reflux disease. Finally, the composition of the reflux
material also has an effect on the development of complications. In a clinical situation, complications of gastroesophageal
reflux disease are more common when there is an alkaline component to the refluxate. In Barrett’s esophagus, the
development of complications such as stricture and ulceration is strongly associated with increased alkaline exposure.

56. Fundamental to understanding disorders of esophageal function is the measurement of the contractility of the
esophageal body and sphincters. Which of the following statement(s) is/are true concerning esophageal manometry
in the investigation of benign esophageal disease?

a.A defective sphincter is predictive of poor long-term response to medical therapy, but a good response to surgery
b.Esophageal manometry can determine the resting pressure and the overall length of the sphincter but not its
abdominal length
c.The LES pressure normally drops to gastric baseline immediately after a swallow before the peristaltic wave
reaches the lower esophagus
d.A Vector Volume below the fifth percentile of normal is the most sensitive measure of mechanical deficiency of the
LES
e.There is no correlation between defects in LES with the severity of gastroesophageal reflux disease
Answer: a, c, d

Esophageal manometry is an investigative tool in which a catheter containing pressure sensors is inserted into the esophagus
and used to measure pressures in esophageal body and sphincters at rest and in response to swallowing. It is indicated in a
number of clinical situations including nonobstructive dysphasia, noncardiac chest pain, and the assessment of
gastroesophageal reflux disease.
The indications for manometry in patients with suspected gastroesophageal reflux are chiefly to assess the status of the LES
and to identify a motility disorder of the body. A defective sphincter is predictive of a poor long-term response to medical
therapy, but a good response to surgery. The presence of a motility defect profoundly alters the operative strategy in patients
with GERD and should always be excluded by manometry prior to operative therapy. In assessment of the LES, three
components are measured: the resting pressure, the overall length of the sphincter, and the abdominal length. A defect in the
values for each of these components are determined when the lower limits of normal (fifth percentile) are determined. A
defect in one or even two components of the LES may be compensated for by good esophageal body function, but when all
three components are defective, excessive esophageal acid exposure is inevitable. All the pressures measured along the
length of the sphincter and around its circumference during the pull-through may be treated as vectors having both
magnitude and direction and hence integrated into a three-dimensional image, the volume (Vector Volume) of which is a
measure of LES resistance. A Vector Volume below the fifth percentile of normal is the most sensitive measure of
mechanical deficiency of the LES. The prevalence of a defective LES increases with increasing severity of GERD, being
the lowest in patients without evidence of endoscopic injury and highest in patients with stricture or Barrett’s esophagus.

57. Which of the following statement(s) is/are true concerning the diagnosis and management of the patient whose
barium esophogram is shown in Figure 18-29?

Asir Surgery MCQs Bank. © 1422H-2002- first impression ©


174
This project was raised after an idia by Dr. Gharama Al-Shehri (consultant surgeon). Developed and typed by Dr. Ghazi Al-Shumrani (intern).
Esophagus, stomach & duodenum
a.The condition is due to neuronal generation of the myenteric plexus in the lower esophageal sphincter
b.The patient will report symptoms of vomiting of sour or bitter material
c.Despite the impressive radiologic picture, passage of the endoscope through the area of narrowing will likely be
possible
d.Manometry and 24 hour pH monitoring should be performed for confirmation of the diagnosis
Answer: c

The x-ray demonstrates moderately advanced achalasia with a dilated esophagus with a narrowed tapering “bird’s beak”
appearance of the distal esophagus. Achalasia is the best known primary motility disorder of the esophagus. It is
characterized by failure of the esophageal body peristalsis and incomplete relaxation of the LES. It is generally thought to
be caused by neuronal degeneration of the myenteric plexus of the esophageal wall, causing aperistalsis, and to loss of
activity of the inhibitory neurons in the LES leading to incomplete relaxation. Patients with achalasia all have dysphagia,
and most have regurgitation. Careful questioning is needed to distinguish the regurgitation from vomiting. Generally, it
occurs during or at the end of a meal, and the material tastes bland rather than sour or bitter. Patients often have to leave the
table to regurgitate, and are usually slow eaters.
Endoscopy frequently reveals residual liquid or food in the esophagus. Unlike a stricture, the narrowing of the lower end
permits the passage of the endoscope, usually with a characteristic “popping” sensation. In every patient with presumed
achalasia, it is very important to view the cardia from below with the endoscope retroflexed, as a small infiltrating
gastroesophageal tumor may otherwise be missed. Manometry is required to establish the diagnosis of achalasia. The classic
features on stationary manometry are: 1) Elevated LES pressure; 2) Incomplete LES relaxation; 3) Absence of esophageal
body peristalsis; and 4) Positive intraesophageal body pressure. Although reports concerning the use of 24 hr pH monitoring
appear in the literature, excessive acid exposure is rare.

58. Which of the following statement(s) is/are true concerning other tests available for investigation of esophageal
disease?

a.A 24 hour pH monitoring is currently the principal method in making the diagnosis of gastroesophageal reflux
disease (GERD)
b.Acid reflux episodes are defined as periods when the esophageal pH is less than 2
c.Twenty-four pH monitoring is only useful in the detection of acid reflux disease
d.The Bernstein test continues to be an important tool in the diagnosis of acid reflux disease
e.Delayed gastric emptying may be an important etiologic factor in patients with GERD
Answer: a, e

The development of 24 hr pH monitoring was a major advance in unraveling the pathophysiology of GERD. It is now the
principal method to make the diagnosis of GERD and has effectively replaced all other methods of measuring esophageal
acid exposure. It is indicated in any patient with symptoms suggesting GERD, unless the symptoms are trivial, or
permanently abolished by a short course of acid suppression therapy. Reflux episodes are defined as periods when the
esophageal pH is less than pH 4. Normal (physiologic) reflux occurs in the form of short rapidly cleared postprandial
episodes. A few episodes of long duration are more injurious than many brief episodes, even though total acid exposure time
may be similar. In addition to the measurement of acid exposure, pH monitoring can also be used to detect excessive
alkaline exposure (pH > 7) in the esophagus. The Bernstein test, in which hydrochloric acid is dripped into the esophagus
via a nasogastric tube, is sometimes used to determine if a patient’s symptoms are reproduced by acidic exposure. It is
basically a measure of esophageal mucosal sensitivity. It has been largely superseded by the use of 24 hr pH monitoring.
Gastric emptying is affected by the composition and consistency of the ingested meal. Delayed gastric emptying may be an
important etiologic factor in patients with GERD and a normal LES.

59. The results for anti-reflux surgery are generally good, however, patients who have failed anti-reflux procedures
constitute a particularly challenging group. Which of the following statement(s) is/are true concerning failed anti-
reflux repairs?

a.A Slipped Nissen is usually the result of an operative technical mistake


b.Disruption of a fundoplication is more prone to occur with a Nissen fundoplication because of the use of the gastric
wall in the repair
c.Postoperative dysphagia in a patient with normal preoperative motility is usually due to a secondary motility
disorder
Asir Surgery MCQs Bank. © 1422H-2002- first impression ©
175
This project was raised after an idia by Dr. Gharama Al-Shehri (consultant surgeon). Developed and typed by Dr. Ghazi Al-Shumrani (intern).
Esophagus, stomach & duodenum
d.Colonic replacement, although technically challenging, usually has superior long-term results when compared to
esophageal replacement with the stomach
Answer: a, d

When patients are correctly selected and the operation performed with conformity with the basic surgical principles, long-
term relief of symptoms is achieved by more than 90% of patients. A number of patterns of failure, however, can occur. The
so-called Slipped Nissen may develop when the upper stomach rides up through the fundoplication, and causes both
dysphagia and heartburn. It is more likely that the condition was created at the time of surgery because the surgeon did not
mobilize the fundus, or because unrecognized esophageal shortening led to inadequate mobilization of the gastroesophageal
junction, causing the surgeon to wrap the stomach around the upper stomach rather than the lower esophagus. Creating too
tight a fundoplication leads to immediate postoperative dysphagia. Manometry shows a high pressure nonrelaxing sphincter
which may be difficult to distinguish from achalasia. Such patients highlight the importance of manometry in all patients
before proceeding with anti-reflux surgery. In a patient with normal preoperative motility, the cause is usually a fault in
technique, and can be prevented by constructing the fundoplication over a 60 F Bougie. Disruption of the fundoplication
that manifests clinically and physiologically by recurrent reflux can be caused by inadequate suture technique, unrecognized
esophageal shortening leading to tension on the wrap, or poor choice of operation. All partial fundoplications, such as the
Toupet procedure are more prone to disruption than a Nissen. This is because the integrity of the repair depends on sutures
to the esophageal wall and not the stomach, and because all these repairs require much more abdominal length of esophagus
than a Nissen, thus placing the repair under tension. Esophagectomy and esophageal replacement are occasionally indicated
in the treatment of advanced GERD. The indications for esophagectomy are Barrett’s esophagus with high grade dysplasia
and what is generally described “burned out esophagus” which includes failure of a third anti-reflux operation, a severe
coexistent motility disorder, or the presence of an undilatable stricture. Either colon or stomach may be used to replace the
esophagus. Colonic replacement is more difficult, requiring three anastomoses rather than one, but it has superior functional
long-term results.

60. A number of diagnostic modalities exist for investigation of structural abnormalities of the esophagus. Which of
the following statement(s) is/are true concerning the use of these investigative studies?

a.Endoscopy should be the first investigation in any patient with foregut symptoms
b.Barrett’s esophagus is suggested when the squamo-columnar junction is more than 2 cm above the
gastroesophageal junction on endoscopic examination
c.There are three areas of esophageal narrowing which can be noted on both barium esophogram and endoscopy
d.The CT appearance of the esophagus is normally a flattened, hollow structure with a thin wall
Answer: b, c, d

Endoscopy is generally the first investigation in patients with foregut symptoms. The exception is when the patient’s chief
complaint is dysphagia, when a “road map” should first be obtained by a barium swallow. The locations of the esophageal
landmarks are measured endoscopically from the incisor teeth. Three landmarks are measured in the region of the cardia:
the level of the crura, the level of the anatomic gastroesophageal junction, and the level of the squamo-columnar junction (Z
line). A hiatal hernia is present when the gastroesophageal junction is more than 2 cm above the crura. Barrett’s esophagus
is suggested when the squamo-columnar junction is more than 2 cm above the gastroesophageal junction but may be
diagnosed if any specialized epithelium is identified above the gastroesophageal junction histologically, regardless of
measured length of the columnar segment. Three areas of esophageal narrowing are frequently noted on both barium
esophogram and endoscopy. The first narrowing is at the site of the cricopharyngeus muscle. The left mainstem bronchus
and aortic arch caused narrowing of the middle third of the esophagus. The most distal narrowing of the esophagus is at the
diaphragmatic hiatus and is caused by the lower esophageal sphincter mechanism. These normal points of narrowing tend to
retard swallowed foreign objects. Also, corrosive liquid ingestion results in prominent mucosal injury at these sites as the
liquid is slowed at passage. CT scan of the esophagus is important in delineating the relationship of esophageal lesions to
adjacent structures, especially the trachea, left main bronchus and aorta. The esophagus normally appears as a flattened
hollow structure with a thin wall. A more circular cross-sectional appearance with a fluid level is evidence of distal
obstruction.

61. Which of the following patient scenarios would be best managed with anti-reflux surgery?

a.A patient with heartburn but normal 24 hour pH monitoring and an intact lower esophageal sphincter
b.A patient with primarily respiratory manifestations of gastroesophageal reflux
Asir Surgery MCQs Bank. © 1422H-2002- first impression ©
176
This project was raised after an idia by Dr. Gharama Al-Shehri (consultant surgeon). Developed and typed by Dr. Ghazi Al-Shumrani (intern).
Esophagus, stomach & duodenum
c.A patient with increased acid exposure and a mechanically defective sphincter who responds well to medical
therapy but requires continued long-term medication for continued relief
d.A patient with gastroesophageal reflux but excessive complaints of epigastric pain, nausea, vomiting, and loss of
appetite
Answer: b, c

The first requirement for consideration of anti-reflux surgery is the objective demonstration of the presence of GERD by 24-
hour pH monitoring. Secondly, the patient must have either symptoms or complications of the disease. Thirdly, the disease
should be caused by defect appropriate to surgical therapy, i.e., a mechanically defective sphincter. Some patients with
increased acid exposure and a mechanically defective sphincter, and who have no complications of the disease respond well
to medical therapy, but they require long-term medication for continued relief. These patients should be given the option of
surgery as a cost effective alternative.
Atypical symptoms of reflux such as respiratory manifestations often respond well to anti-reflux surgery. When respiratory
symptoms are combined with typical symptoms such as heartburn and regurgitation, the results of anti-reflux surgery are
generally good.
Complaints of epigastric pain, nausea, vomiting, and loss of appetite may be due to excessive duodenogastric reflux which
occurs in about 11% of patients with gastroesophageal reflux disease. This problem is usually, but not invariably, confined
to patients who have previous upper gastrointestinal surgery. The coexistence of these gastric symptoms in a patient who
also has typical symptoms of GERD should prompt a thorough evaluation of the stomach using a bile probe, 24 hour pH
monitoring or radionucleotide scanning. In such patients, the correction of only the incompetent cardia can result in a
disgruntled individual who continues to complain of nausea and epigastric pain on eating.

62. Which of the following statement(s) concerning pharyngoesophageal disorders is/are true?

a.In neuromuscular diseases, dysphagia is often worse for liquids than for solids
b.Cricomyotomy may be indicated for a wide variety of neuromuscular disorders involving the pharyngoesophageal
phase of swallowing
c.Excision of a Zenker’s diverticulum is indicated to prevent malignant change in the sac
d.Complications of all operations on the cervical esophagus include hematoma formation and recurrent nerve
paralysis
Answer: a, b, d

Disorders of the pharyngoesophageal phase of swallowing result from a discoordination of the neuromuscular events
involved in chewing, initiation of swallowing, and propulsion of the material from the oropharynx to the cervical
esophagus. The commonest causes of pharyngoesophageal dysphagia are neuromuscular diseases. The most important are
cerebrovascular disease, myasthenia gravis, Parkinson’s disease, multiple sclerosis and muscular diseases such as myotonic
dystrophy and polymyositis. In neuromuscular diseases, dysphagia is often worse for liquids than for solids. Choking,
repetitive pneumonia, nasal regurgitation and hoarseness are also prominent features. The surgeon’s role in the treatment of
cricopharyngeal disorders is to reduce outflow resistance by performing a cricomyotomy. Initially this was recommended
only for patients with demonstrable failure of the upper esophageal sphincter relaxation. More recently, a number of reports
indicate a wide variety of neuromuscular diseases that may be improved by cricomyotomy. The surgical options in Zenker’s
diverticulum are either excision or suspension. Excision is sometimes recommended on the grounds that malignant change
in the sac is prevented, but there is no evidence that excision carries any greater protective role than suspension, which
effectively prevents stagnation of food material, thus removing the presumed cause of malignant change. Suspension also
removes the risk of contamination of the operative site, the risk of subsequent breakdown of the closure site with fistula
formation, and the risk of narrowing of the esophagus. In either case, recurrence is likely if cricomyotomy is not performed,
because the underlying defect which predisposes to the diverticulum persists. All operations on the cervical esophagus carry
the risk of hematoma formation and recurrent nerve paralysis. The venous pumping action of the lung can cause the
development of a large hematoma in the mediastinum postoperatively, therefore meticulous hemostasis is critical for the
performance of this operation.

63. Barrett’s esophagus is a complication of gastroesophageal reflux disease. Which of the following statement(s)
is/are true concerning this condition?

a.The histologic hallmark is the presence of “specialized” columnar epithelium regardless of how far it extends into
the esophagus
Asir Surgery MCQs Bank. © 1422H-2002- first impression ©
177
This project was raised after an idia by Dr. Gharama Al-Shehri (consultant surgeon). Developed and typed by Dr. Ghazi Al-Shumrani (intern).
Esophagus, stomach & duodenum
b.Barrett’s epithelium will frequently regress with medical therapy or anti-reflux surgery
c.High grade dysplasia will frequently be associated with foci of invasive carcinoma
d.Patients with adenocarcinoma arising in Barrett’s esophagus have a high incidence of p53 gene mutations
Answer: a, c, d

Barrett’s esophagus is now recognized as a complication of advanced gastroesophageal reflux disease. The histologic
hallmark of Barrett’s esophagus is the presence of “specialized” columnar epithelium, which shows features of intestinal
metaplasia, easily recognized by the presence of goblet cells. The presence of specialized epithelium is now regarded as the
pathonomonic feature of Barrett’s esophagus regardless of how high it extends into the esophagus. Barrett’s esophagus may
exist on its own, or may be itself associated with ulceration, stricture, and malignant change. Once Barrett’s epithelium is
present, medical therapy or anti-reflux surgery rarely causes it to regress. Unless it is actually ablated, for example with
laser therapy, it persists. The most significant feature of Barrett’s esophagus is its malignant potential. The metaplastic
epithelium usually undergoes dysplastic change prior to becoming frankly neoplastic. High grade dysplagia is synonymous
with carcinoma in situ, and if the esophagus is removed for such a condition, up to 50% will demonstrate foci of invasive
carcinoma.
In the past, the pathophysiology of Barrett’s esophagus was associated with alkaline reflux on esophageal pH monitoring.
However, more recently using a bile sensor for monitoring bilirubin, this condition is frequently associated with excessive
bile in the esophagus. Repetitive injury from noxious gastric juice can lead during the repair process to mutations in the p53
gene. Patients with adenocarcinoma arising in Barrett’s esophagus have a high incidence of p53 mutations.

64. Which of the following statement(s) is/are true concerning the blood supply and lymphatic drainage of the
esophagus?

a.The thoracic esophagus receives no direct branches from the aorta therefore allowing the technique of transhiatal
(blunt) esophagectomy
b.Bleeding esophageal varices are most prominent in the mid-esophagus
c.Lymphatic drainage of the lower third of the esophagus goes entirely to the abdominal lymphatic system
d.Nodal involvement in esophageal cancer is quite common even if the tumor is limited to the level of the submucosa
Answer: d

The blood supply and venous drainage of the esophagus are largely segmental. The inferior thyroid artery provides the main
blood supply to the cervical portion of the esophagus. The thoracic portion of the esophagus receives its blood supply from
two sources; branches from two or three bronchial arteries provide the proximal arterial supply and branches directly from
the aorta supply the more distal thoracic esophagus. Intrathoracic mobilization of the esophagus during performance of anti-
reflux procedures often require ligation of these branches. The venous plexus in the submucosa collects capillary blood and
delivers it into a periesophageal venous plexus. The left gastric vein or coronary vein provides the principal collateral in
portal hypertension when esophageal varices develop. The submucosal veins become much more superficial in the most
distal esophagus, 1–2 cm above the gastroesophageal junction, and are consequently the most common site of bleeding in
portal hypertension.
The lymphatics of the esophagus form a rich submucosal network draining into regional lymph nodes in the periesophageal
connective tissue. There is thus little barrier to longitudinal spread of cancer in the esophagus. Lymphatic drainage from the
upper two-thirds of the esophagus is usually cephalad, but drainage from the lower one-third is in both directions. Although
lymphatic metastasis in the esophagus generally involve the regional lymph nodes in proximity, nodal involvement may
occur several centimeters away from the primary lesion because of the rich intramural lymphatic anastomotic channels.
When a carcinoma is limited to the mucosa, the incidence of lymphatic metastases is low, but once into the submucosa, the
incidence rises to 60%.

65. Which of the following statement(s) is/are true concerning the process of swallowing and esophageal transit of
food?

a.Injury to the recurrent laryngeal nerves can cause motility problems of the cervical esophagus and resulting
aspiration
b.Esophageal reflux does not lead to impaired esophageal motility
c.Relaxation of the LES is mediated via inhibitory neurons
d.The overall length of the LES is the only factor influencing the pressure gradient of the sphincter
e.A mechanically defective sphincter is always associated with increased esophageal acid exposure
Asir Surgery MCQs Bank. © 1422H-2002- first impression ©
178
This project was raised after an idia by Dr. Gharama Al-Shehri (consultant surgeon). Developed and typed by Dr. Ghazi Al-Shumrani (intern).
Esophagus, stomach & duodenum
Answer: a, c

The cricopharyngeus muscle is a continuation of the inferior constrictor of the pharynx and receives its innervation via both
the right and left recurrent laryngeal nerves. Although much attention is given to vocal cord dysfunction that accompanies
recurrent laryngeal nerve damage, it is clear that cricopharyngeal sphincter dysfunction and motility problems of the
cervical esophagus can occur with injury to these nerves. Serious aspiration following recurrent nerve injury is caused not
only by the cricopharyngeal dysfunction, but also by additional morbidity incurred because of the inability to close the
glottis during swallowing and loss of the protection afforded by effective coughing. Clinically, peristaltic defects of the
esophageal body fall into one of to broad categories. One category is characterized by a defect in organization of peristaltic
waves, and is primarily a neural phenomenon. The other notable defect is reduction of the power (amplitude) of peristalsis
and is usually due to muscle damage secondary to severe reflux or replacement with fibrous tissue as happens in
scleroderma and other connective tissue diseases or with severe reflux. The LES provides a pressure barrier between the
esophagus and stomach. The sphincter normally remains actively closed to prevent reflux of gastric contents into the
esophagus. Relaxation of the LES is mediated by inhibitory neurons. It occurs either to allow entry of food, or to allow exit
of air during belching. The ability of the LES to remain closed in the face of a pressure gradient tending to promote reflux
of gastric contents from the positive pressure environment of the stomach into the negative pressure environment of the
chest depends on several features. The most significant is the resting pressure. However, of equal importance is the ability
of the LES to respond to variations in intra-abdominal pressure associated with daily activities. Such elevations would
normally be transmitted to the sphincter, causing it to collapse and remain closed, provided sufficient length of the sphincter
remains exposed to the abdominal pressure and the compressive effect of the crura. The abdominal length is often reduced
in hiatal herniation, because of attenuation of the pharyngoesophageal membrane. The overall length of the LES is also an
important determinant of competence, much as the total resistance of a series of resisters in a circuit is the sum of the
individual resistances. A mechanically-defective sphincter, however, is not always associated with increased esophageal acid
exposure because it may be compensated by the clearance function of the esophageal body. The role of the esophageal body
in limiting acid reflux is related to its ability to clear the esophagus of acid. This clearance has two components: volume
clearance which requires peristalsis, and chemical clearance which requires saliva.

66. Which of the following statement(s) is/are true concerning the management of this patient?

a.The risk of perforation of the esophagus associated with balloon dilatation may be as high as 10%
b.An anti-reflux procedure should be universally performed for any operative myotomy
c.Successful relief of dysphagia can be achieved in up to 90% of patients with a single pneumatic dilatation
d.Thoracoscopic myotomy is associated with significantly poorer results than the open procedure
e.Prospective randomized studies and retrospective data appear to support a surgical approach for achalasia
Answer: a, e

The mainstay of treatment in achalasia is either balloon dilatation or surgery. Balloon dilatation has an advantage that it can
be performed as an outpatient and has minimal recovery time. It is less likely to be effective than surgical treatment, and
frequently needs to be repeated. The risk of perforation of the lower esophagus is higher with this procedure than with any
other form of esophageal instrumentation and varies from 2–10%. The risk of gastroesophageal reflux following dilatation
is not known, but symptomatically the risk appears to be low.
All surgical procedures employ a variant of Heller’s myotomy, in which the circular muscle of the lower esophagus is
divided. In the United States, most myotomies are carried out through the chest, but the abdominal approach is favored in
Europe. Regardless of the route chosen, four key principles are important, namely: 1) adequate myotomy, 2) minimal hiatal
disturbance, 3) anti-reflux protection without creation of obstruction, and 4) prevention of rehealing. The advent of
minimally invasive surgery has led to the development of thorascopic and laparoscopic myotomy, and these are now being
extensively performed with comparable results to open surgery. There is broad agreement that if a myotomy is performed
through the abdomen, an anti-reflux procedure should be added, and that a full Nissen wrap, however floppy, leads to long-
term failure. When approached through the chest, there is controversy about the need for an anti-reflux procedure, as it is
claimed that less hiatal disturbance and more limited myotomy is possible by this route. Thoracoscopic myotomy, with
enhanced view, enables a more precise determination of the distal myotomy and therefore may not require a anti-reflux
procedure.
A single pneumatic dilatation achieves adequate relief of dysphasia and pharyngeal regurgitation in about 60% of patients.
Repetitive dilatations increase this figure to about 70%. Only one controlled randomized study comparing the two modes of
therapy has ever been performed. The results of this study as well as a number of retrospective studies would appear to
support operative myotomy as the initial treatment of choice.

Asir Surgery MCQs Bank. © 1422H-2002- first impression ©


179
This project was raised after an idia by Dr. Gharama Al-Shehri (consultant surgeon). Developed and typed by Dr. Ghazi Al-Shumrani (intern).
Esophagus, stomach & duodenum

67. Which of the following statement(s) is/are true concerning the surgical anatomy of the esophagus?

a.Surgical exposure of the cervical esophagus is best gained via the right neck
b.Spontaneous esophageal perforation tends to be associated with leakage into the left chest
c.Access to the entire thoracic esophagus can be obtained only via the left chest
d.The lower esophageal sphincter can be recognized distinctly by inspection of the gastroesophageal junction
Answer: b

A detailed knowledge of the relations of the esophagus is essential for the surgeon to be able to identify the site and
significance of lesions seen by indirect studies such as endoscopy, contrast radiography, and CT scanning, as well as the safe
performance of surgical procedures. The cervical esophagus is about 5 cm long. It begins at the level of C6 and extends to
the lower border of T1, curving slightly to the left in its descent. Consequently, although the surgical approach to this
portion of the esophagus may be from either side of the neck through an incision along the anterior border of the
sternocleidomastoid muscle, the left side is chosen if possible. Above the level of the tracheal bifurcation, the esophagus
moves to the right of the descending aorta. It then moves to the left, passes behind the tracheal bifurcation and the left main
bronchus and descends to the diaphragm. In the lower third, the esophagus courses anteriorly and to the left to pass through
the diaphragmatic hiatus. The lower esophagus is covered only by a flimsy mediastinal pleura on the left, and it is this
portion which is most commonly the site of spontaneous perforation in Boerhaave’s syndrome. In general, the lower
esophagus is most easily approached through the left chest, but access to the supra-aortic esophagus is restricted. Thus, a
left thoracotomy is most useful for performing procedures involving the lower esophagus. However, access to the entire
thoracic esophagus can be obtained only from the right chest. This incision, however, limits access to intraabdominal organs
by the position of the liver and therefore normally requires a separate upper abdominal incision. The abdominal esophagus
begins as the esophagus enters the abdomen through the diaphragmatic hiatus. It is surrounded by a fibroelastic membrane,
the phrenoesophageal ligament which arises from the subdiaphragmatic fascia. The lower limit of the pharyngoesophageal
membrane anteriorly is marked by a prominent fat pad, which corresponds to the gastroesophageal junction. The lower
esophageal sphincter (LES) is a zone of high pressure 3–5 cm long at the lower end of the esophagus. Although it does not
correspond to any macroscopic anatomical structure, its function appears to be related to the microscopic architecture of the
muscle fibers.

68. Which of the following statement(s) is/are correct concerning the patient whose barium esophogram is shown
below?

a.The patient’s complaint would be primarily chest pain and to a lesser degree dysphagia
b.The pathognomic feature of manometry is the presence of prolonged high amplitude waves
c.The patient will likely experience nutritional problems
d.The first line of treatment for this patient is surgical myotomy
Answer: a, b

The barium esophogram of these two patients shows diffuse esophageal spasm resulting in a cork screw esophagus with
multiple contractions. See Fig. 18-35. These primary motor disorders are characterized by substernal chest pain. In the
nutcracker variety, as demonstrated in this x-ray, the pain is central crushing pain with no relation to food ingestion and
differs from angina in that it more frequently comes on at rest. Dysphagia or classic heartburn may be present but tend to be
overshadowed by the chest pain. Barium radiography and endoscopy are generally not helpful. The pathognomic feature of
manometry is the presence of prolonged high amplitude waves, with a peak greater than 180 mm Hg. Diffuse esophageal
spasm and nutcracker esophagus are benign conditions which rarely cause nutritional problems and do not lead to life-
threatening complications. For this reason, symptom control is the only significant goal of treatment. Medical treatment for
diffuse esophageal spasm and nutcracker esophagus is focused on abolishing strong simultaneous contractions and generally
employs calcium channel blocking agents or long-acting nitrates. Surgery for these conditions are generally only considered
when medical treatment is ineffective.

69. Which of the following statement(s) is/are true concerning tracheoesophageal fistulas?

a.The majority of acquired tracheoesophageal fistulas are due to malignant disease


b.A water-soluble contrast esophogram should be obtained for diagnosis
c.Malignant tracheoesophageal fistulas represent one of the few indications for an endoesophageal prosthesis
Asir Surgery MCQs Bank. © 1422H-2002- first impression ©
180
This project was raised after an idia by Dr. Gharama Al-Shehri (consultant surgeon). Developed and typed by Dr. Ghazi Al-Shumrani (intern).
Esophagus, stomach & duodenum
d.A benign tracheoesophageal fistula from an endotracheal intubation injury often requires a thoracotomy for repair
Answer: a, c

Ninety percent of acquired fistulas between the esophagus and tracheobronchial tree in adults are the result of malignant
disease. Tracheoesophageal fistulas complicate the course of disease in about 5% of patients who have esophageal
carcinoma. Nearly 80% of patients with malignant tracheoesophageal fistulas die within three months of the onset of
symptoms and in 85% of these patients, the cause of death is aspiration pneumonia, not distant metastatic disease. For the
most part, malignant tracheoesophageal fistula represents incurable disease for which resection carries significant mortality
and is seldom indicated. Palliative relief of recurrent aspiration is the aim of therapy. Effective occlusion of the fistula may
be achieved by insertion of one of a variety of available endoesophageal endoprostheses. These tubes are placed into the
esophagus with the aid of an esophagoscope and may occlude the esophageal side of the fistula sufficiently to allow
swallowing of liquids without aspiration into the tracheobronchial tree. More recently, expandable metal stents have been
used successfully in the treatment of malignant tracheoesophageal fistulas.
Nonmalignant fistulas result from the erosion by contiguous infected subcarinal mediastinal lymph nodes; trauma; late
sequelae of chronic mid-esophageal traction diverticulum; or erosion by an endotracheal or tracheostomy tube cuff in a
patient requiring prolonged ventilatory support. Small fistulas, such as resulting from an endotracheal intubation injury, are
approached through a cervical collar or oblique incision anterior to the sternocleidomastoid muscle. Although such cuff
injuries usually produce circumferential tracheal damage which necessitates a tracheal resection, this can also be performed
through a cervical collar incision.

70. Esophageal cysts arise as outpouchings of the embryonic foregut. Which of the following statement(s) is/are true
concerning esophageal cysts?

a.The cyst lining will be lined only by stratified squamous epithelium


b.Most esophageal cysts cause symptoms in the first year of life
c.An asymptomatic esophageal cyst can be managed conservatively
d.The diagnosis of an esophageal cyst is usually made radiographically
Answer: b, d

Embryologically, the esophagus is lined by simple columnar ciliated epithelium, which is eventually replaced by stratified
squamous epithelium. The esophageal cyst may therefore contain both of these types of epithelium as well as fat and
smooth muscle. The esophageal duplication cyst is a variation of the foregut cyst, extends along the length of the thoracic
esophagus, and is lined by squamous epithelium. More than 60% of esophageal cysts cause either respiratory or esophageal
symptoms in the first year of life. Those located in the upper third of the esophagus tend to present in infancy, while the
lower-third cyst may be asymptomatic initially and present later in childhood. Adults present with dysphasia, choking,
retrosternal pain when previously asymptomatic cysts enlarge as the result of bleeding or infection. The diagnosis of an
esophageal cyst can usually be made on the basis of atypical radiographic appearance. The PA and lateral chest x-ray,
barium esophogram, and in some cases a CT scan, will confirm the diagnosis in almost all patients. Because esophageal
cysts have a predilection for bleeding, ulceration, perforation, and infection, excision is generally recommended. This can
generally be achieved with low morbidity by an extramucosal resection.

71. Which of the following statement(s) is/are true concerning infectious esophagitis?

a.Candida albicans is not normally found in the mouth but results from the overgrowth of this fungus in patients on
broad spectrum antibiotics
b.Candida esophagitis is usually self-limited and is seldom associated with chronic problems
c.Systemic therapy is seldom indicated
d.Small ulcers on barium esophogram in a transplant patient complaining of dysphagia and odynophagia are likely
due to herpes simplex viral infection
Answer: d

Chronic debilitation, immunosuppression, and prolonged use of antibiotics predisposes the development of infectious
esophagitis with candida albicans being the most common cause. Candida albicans is a fungus that normally is a commensal
inhabitant of the mouth, oral pharynx, and GI tract. This fungus may become pathogenic in patients who are severely
debilitated or immunosuppressed. In recent years, the use of broad spectrum antibiotics, immunosuppression in organ
transplant patients, and the wide use of chemotherapeutic agents have resulted in an increased number of patients with
Asir Surgery MCQs Bank. © 1422H-2002- first impression ©
181
This project was raised after an idia by Dr. Gharama Al-Shehri (consultant surgeon). Developed and typed by Dr. Ghazi Al-Shumrani (intern).
Esophagus, stomach & duodenum
monilial esophagitis. As the disease progresses, transmural invasion of the esophageal wall occurs. Although the esophagitis
can be controlled with antifungal therapy, if the patient survives the underlying illness, chronic stricture formation may
result after healing. Minimally compromised patients with mild monilial esophagitis should receive oral nystatin suspension
as a primary treatment. More immunosuppressed patients or those with severe cases warrant high doses of fluconazole and
ketoconazole. Intravenous fluconazole or amphotericin B are utilized in granulocytopenic patients.
Viral esophagitis is the second most common cause of infectious esophagitis. Herpes simplex viral infection is the most
common infection in the immunosuppressed transplant patient. Characteristically, viral esophagitis produces mucosal
ulceration with patients presenting with dysphasia and odynophagia. The esophageal ulcers are characteristically small (<
1.5 cm). The diagnosis is established endoscopically by biopsy, brushings, and washings for cytology, histology, and viral
culture. The infection usually responds well to treatment with acyclovir.

72. Which of the following statement(s) is/are true concerning the pathology of squamous cell carcinoma of the
esophagus?

a.Carcinoma in situ will gradually progress to invasive squamous cell carcinoma over a period of two to four years
b.The most common location for squamous cell carcinoma of the esophagus is the upper and mid-thoracic segment
c.Esophageal carcinoma tends to be multifocal
d.Macroscopically, ulcerative lesions with extensive infiltration of the adjacent esophageal wall are most common
e.Lymph node metastases are present in at least 75% of patients at the time of initial diagnosis
Answer: a, b, d, e

Pathologically, esophageal carcinoma occurs over a spectrum that ranges from the early lesion (carcinoma in situ), which is
limited to the mucosa, to the more advanced form, in which the tumor penetrates the muscle layers of the esophagus and
beyond. Carcinoma in situ typically is found in patients between 40 and 50 years of age and gradually progresses to
invasive squamous cell carcinoma over two to four years. Using the arbitrary division of the esophagus, 8% of squamous
cell carcinomas occur in the cervical esophagus, 55% in the upper and mid-thoracic segments, and 37% in the lower
thoracic segment which extends to the GE junction. Macroscopically, 60% of squamous cell carcinomas of the esophagus
are fungating intraluminal growths, 25% of ulcerative lesions are associated with extensive infiltration of the adjacent
esophageal wall, and 15% are infiltrating. Esophageal carcinoma tends to be multi-focal, and a patient who survives
treatment of one carcinoma has at least twice the risk of developing a second primary esophageal neoplasm than the normal
population.
Esophageal carcinoma is notorious for its aggressive biologic behavior. Mediastinal, supraclavicular, or celiac lymph node
metastases are present in at least 75% of patients with esophageal cancer at the time of initial diagnosis. Unfortunately,
when lymph node metastases are present, five-year survival is only 3%, compared with 42% when there is no lymph node
spread.

73. A 54-year-old woman experiences pain in both the anterior and posterior left chest and the epigastrium following
balloon dilatation performed for achalasia. Which of the following statement(s) is/are true concerning this patient’s
diagnosis and management?

a.A normal chest x-ray will rule out an esophageal perforation


b.Barium should never be used in performance of a contrast study with a diagnosis of esophageal perforation
c.Conservative, nonoperative treatment may be indicated
d.If surgical repair is necessary, the patient should undergo esophagomyotomy and a partial gastric fundoplication
Answer: c, d

It is axiomatic that pain or fever after esophageal instrumentation or operation is indicative of an esophageal perforation
until proven otherwise. It is an indication for immediate contrast esophogram. Because the morbidity and mortality rates
associated with esophageal perforation are directly related to the time interval between diagnosis of the injury and its repair
or drainage, an aggressive attitude toward diagnosing the perforation must be adopted. When the diagnosis is considered, a
water-soluble contrast agent should be administered. If this study is negative, dilute barium should be administered. Barium
is relatively inert, and the fear of barium extravasating in the mediastinum through the site of injury and producing a severe
reactive mediastinitis is unfounded. A chest x-ray may help confirm the diagnosis by demonstrating air in the soft tissues of
the neck or mediastinum or a hyrdo-or pneumothorax. A normal chest x-ray, however, does not rule out an esophageal
perforation.

Asir Surgery MCQs Bank. © 1422H-2002- first impression ©


182
This project was raised after an idia by Dr. Gharama Al-Shehri (consultant surgeon). Developed and typed by Dr. Ghazi Al-Shumrani (intern).
Esophagus, stomach & duodenum
Although most esophageal perforations require operative intervention, selected patients may be managed nonoperatively
with cessation of oral intake, administration of antibiotics, and intravenous hydration until the disruption heals or the small
contained cavity begins to decrease in size. The usual clinical settings for such perforations that are encountered are cervical
esophageal tears caused by esophagoscopy; intramural dissection that has occurred during dilatation of a stricture or
pneumatic dilatation for achalasia, and an asymptomatic esophageal anastomotic disruption discovered on a routine
postoperative contrast study. Perforations complicating pneumatic dilatation for achalasia occur in 4% to 6% of patients,
and most are small and well managed medically with antibiotics and intravenous hyperalimentation. If operation is required
for suture repair of the perforation, an esophagomyotomy to relieve the distal obstruction, and a partial fundoplication to
buttress the tear should be performed if possible.

74. Which of the following statement(s) is/are correct concerning the diagnostic studies for esophageal carcinoma?

a.A chest and upper abdominal CT scan is useful for both staging and predicting resectability
b.A barium swallow is an unnecessary test in a patient with dysphagia
c.Bronchoscopy should be performed in all patients with carcinoma of the upper and middle thirds of the esophagus
d.Bone and brain scans should be obtained routinely to rule out distant metastasis
e.Endoscopic ultrasound is a potentially sensitive examination for the staging of esophageal cancer
Answer: c, e

A barium swallow examination is the first study that should be obtained in a patient who complains of dysphagia. Tumors of
the cervical esophagus are difficult to identify by barium swallow examination and carcinoma of the cardia may be
confused with achalasia or esophageal spasm. Nevertheless, the barium swallow examination localizes obvious esophageal
pathology in preparation for subsequent esophagoscopy and allows the endoscopist to predict the level at which the tumor is
located and the area which requires the most careful examination. The chest and upper abdominal CT scan is now the
standard radiographic technique for staging esophageal carcinoma. Esophageal wall thickness, regional adenopathy or
pulmonary, liver, adrenal or distant nodal metastasis can be identified. Although CT is suggested to have a role in evaluating
resectability of esophageal carcinoma, it is particularly limited in its ability to detect invasion of the gastric cardia or aortic
invasion. Bone scan is not warranted unless the patient has specific complaints suggesting that bone metastases exists.
Similarly, routine brain scans are not indicated as brain metastases from carcinoma of the esophagus are uncommon.
Bronchoscopy should be performed in patients with carcinoma of the upper and middle thirds of the esophagus to exclude
invasion of the posterior membranous trachea or mainstem bronchi, which precludes a safe esophagectomy. Endoscopic
ultrasound is being used with increasing frequency as an adjunct to the standard radiologic and endoscopic assessment of
esophageal disease. It offers the potential for more sensitive staging of esophageal carcinoma by detecting the depth of
invasion and the presence of abnormal mediastinal adenopathy.

75. Which of the following conditions are associated with the development of esophageal carcinoma?

a.Caustic esophageal stricture


b.Achalasia of the esophagus
c.Plummer-Vinson syndrome
d.Esophageal diverticula
Answer: a, b, c, d

Chronic irritation of the esophageal mucosa by a variety of noxious stimuli (alcohol, tobacco, hot foods and liquids)
eventually may lead to the development of esophageal carcinoma. A variety of other esophageal lesions have a recognized
premalignant nature. The patient who survives the initial injury long enough to develop a caustic esophageal stricture has a
1000-fold increased risk of developing carcinoma compared with the normal population. Ten to 12% of patients with
achalasia of the esophagus who are observed 15 years or more develop esophageal carcinoma. This is thought to be related
to the irritating effects of the fermenting intraesophageal contents on the adjacent esophageal mucosa. Plummer-Vinson
syndrome is a premalignant esophageal condition. Patients with this syndrome are typically elderly women who have
cervical dysphasia and iron deficiency anemia. About 10% of patients will develop squamous cell carcinoma of the
hypopharynx, oral cavity or esophagus. Finally, there have been isolated reports of esophageal carcinomas found
incidentally within esophageal diverticula, presumably as the result of the irritating effects on the mucosa of stagnant,
putrefying food within the pouch. Esophageal diverticula are therefore regarded as premalignant esophageal lesions
although this occurrence is extremely rare.

Asir Surgery MCQs Bank. © 1422H-2002- first impression ©


183
This project was raised after an idia by Dr. Gharama Al-Shehri (consultant surgeon). Developed and typed by Dr. Ghazi Al-Shumrani (intern).
Esophagus, stomach & duodenum

76. Which of the following statement(s) is/are correct concerning the options for resection of esophageal carcinoma?

a.The development of reflux esophagitis seldom occurs following intrathoracic resection due to the limited life
expectancy of these patients
b.Transhiatal esophagectomy, although conceptually sound, is not technically possible in most patients with esophageal
carcinoma
c.Transhiatal resection, although less morbid, has unfavorable survival statistics compared to transthoracic resection
d.Radical transthoracic esophagectomy with en bloc dissection of continuous lymph node bearing tissues has not been
shown to improve survival over transhiatal esophagectomy
Answer: d

For most patients with localized esophageal carcinoma, resection provides the most effective and reliable palliation of
dysphagia. The rational surgical approach to distal esophageal carcinoma has been a left thoracoabdominal incision. Tumors
involving the mid-esophagus have been resected either through a thoracoabdominal or separate thoracic and abdominal
incision, and a high thoracic esophagogastric anastomosis is performed. The major disadvantages of this technique are the
necessity of a thoracotomy in debilitated patients with esophageal obstruction as well as the disastrous complications
following intrathoracic esophageal anastomotic leak. Although recent results have shown improved operative mortality
rates, the operation can still be associated with significant morbidity and mortality. A further disadvantage of the standard
intrathoracic esophagogastric anastomosis is inadequate long-term relief of dysphagia either due to tumor recurrence at the
anastomotic suture line or due to the development of reflux esophagitis above the anastomosis. Although it has been long
taught that the patient with esophageal carcinoma does not live long enough to develop reflux esophagitis after a low
intrathoracic esophagogastric anastomosis, this is clearly not the case, and the development of reflux in these patients can
produce not only severe pyrosis and reflux symptoms, but also dysphagia from benign stenosis.
During the last two decades, the technique of transhiatal esophagectomy without thoracotomy has been popularized as an
operation that minimizes the factors responsible for poor results from traditional transthoracic esophageal resection and
reconstruction. In experienced hands, transhiatal esophagectomy without thoracotomy is possible in over 90% of patients.
The survival data is comparable to those obtained in most series of transthoracic resection with results usually
demonstrating decreased postoperative morbidity and mortality. Although conceptually radical transthoracic esophagectomy
with en bloc dissection of contiguous lymph node bearing tissues would appear to offer a better “cancer operation” than
transhiatal esophagectomy with no formal lymph node dissection, current survival results are not statistically different.
These data suggest that survival after resection for esophageal carcinoma is more a function of the extent and stage of the
tumor rather than the size of the specimen or the number of lymph nodes removed.

77.The incidence of adenocarcinoma of the esophagus is increasing at a very rapid rate, which is largely the result of
the growing prevalence of adenocarcinoma arising in Barrett’s mucosa. Which of the following statement(s) is/are true
concerning adenocarcinoma of the esophagus?

a. Barrett’s mucosa with specialized columnar epithelium characterized by veliform folds, lined by secreting
columnar and goblet cells has the highest association with carcinoma of the esophagus
b. Less than 5% of patients with Barrett’s mucosa will harbor adenocarcinoma
c. Severe dysplasia of Barrett’s mucosa requires frequent reexamination and biopsy
d. Adenocarcinoma of the esophagus has a less aggressive behavior than squamous cell carcinoma
Answer: a

It is estimated that patients with Barrett’s esophagus are 40 times more likely to develop adenocarcinoma than the general
population. The true incidence of Barrett’s esophagus in the general population is unknown, but it is estimated that
adenocarcinoma arises in up to 8% to 15% of patients with columnar epithelium lined esophagus. Of the three characteristic
histologic patterns for Barrett’s mucosa, the specialized or intestinal type of metaplasia which is characterized by veliform
folds lined by a single layer of glycoprotein secreting columnar cell and mucous-secreting goblet cells has the highest
association with carcinoma. Dysplasia occurs to varying degrees in Barrett’s mucosa and is clearly a premalignant lesion.
Severe dysplasia is almost always associated with carcinoma in situ and mandates aggressive therapy. As is true of
squamous cell carcinomas, esophageal adenocarcinoma has an aggressive biologic behavior that is characterized by frequent
transmural invasion and lymphatic spread.

78.Which of the following statement(s) is/are true concerning esophageal diverticula?


Asir Surgery MCQs Bank. © 1422H-2002- first impression ©
184
This project was raised after an idia by Dr. Gharama Al-Shehri (consultant surgeon). Developed and typed by Dr. Ghazi Al-Shumrani (intern).
Esophagus, stomach & duodenum

a. A Zenker’s diverticulum characteristically occurs in older patients


b. Mediastinal granulomatous disease usually results in a mid-esophageal traction diverticulum which is usually
asymptomatic
c. An epiphrenic diverticulum that presents to the right of the esophagus should be managed via left thoracotomy
d. Minimally symptomatic epiphrenic diverticula should not be operated upon
Answer: b, c, d

An esophageal diverticulum is an epithelial-lined mucosal pouch that protrudes from the esophageal lumen. Most
esophageal diverticula are acquired, and occur predominantly in adults. The pharyngoesophageal (Zenker’s diverticulum) is
the most common esophageal diverticulum and typically occurs in patients between 30 and 50 years of age. Mediastinal
granulomatous disease (e.g., tuberculosis or histoplasmosis) is the most common cause of mid-esophageal traction
diverticulum. This type of diverticulum is much smaller than the pulsion diverticulum and has a characteristic blunt tapered
tip that points toward the adjacent subcarinal or peribronchial lymph nodes to which it adheres. It is typically diagnosed as
an incidental finding on a barium esophogram and is almost always asymptomatic. No specific treatment is indicated.
An epiphrenic or supradiaphragmatic diverticulum occurs within the distal 10 cm of the thoracic esophagus as a pulsion
diverticulum that arises because of abnormally elevated intraluminal esophageal pressure. Although many patients are
asymptomatic at the time of diagnosis on barium esophogram, others have symptoms from the frequently associated
esophageal conditions: hiatal hernia, diffuse esophageal spasm, achalasia, reflux esophagitis and carcinoma. Pouches
smaller than 3 cm and causing little or no symptoms require no treatment. Severe dysphagia, chest pain, or an anatomically
dependent or enlarging pouch are indications for repair. The surgical approach to an epiphrenic diverticula is through a left
6th or 7th interspace posterolateral thoracotomy. This is the case even for diverticula that present to the right of the
esophagus.

79.Which of the following statement(s) is/are true concerning caustic injury to the esophagus?

a. Alkaline injury is more destructive than acid injury


b. Acid ingestion is not injurious to the stomach due to its normal acidic pH
c. Ingested caustic agents rapidly pass through the esophagus and stomach into the small intestine
d. Unless perforation occurs, clinical manifestations resolve quickly with initial clinical improvement noted
e. Children are less likely to form a late esophageal stricture than adults
Answer: a, d, e

Caustic injury occurs in two broad categories of patients, children younger than 5 years of age who accidentally swallow
these agents, and adults who are attempting suicide. The most common agents responsible for caustic esophageal injuries
are alkalis, acids, bleach and detergents. Ingestion of detergents and bleaches virtually always cause only mild esophageal
irritation which heals without significant adverse sequelae. Acids and alkalis may have devastating effects which range from
acute multi-organ necrosis and perforation to chronic esophageal and gastric strictures. Alkalis are more destructive,
producing liquefaction and necrosis which almost insures deep penetration whereas acids usually cause coagulation necrosis
that, in part, limits the depth of injury. Liquid alkali preparations have prolonged contact with the mucosa of the esophagus
and stomach due to its high viscosity. In addition, ingested acids typically pass through the esophagus quickly, producing
major gastric injury with relative sparing of the esophagus. In response to either ingested acid or alkali, reflex pyloric spasm
occurs, with resultant pooling of these agents in the gastric antrum.
The clinical manifestations of caustic ingestion are directly related to the amount and character of the agent ingested. When
esophageal or gastric perforation results from caustic ingestion, patients demonstrate progressive severe sepsis and
hypovolemic shock until appropriate resuscitative measures are instituted. In the absence of gastric or esophageal
perforation, the acute clinical manifestations typically resolve within days, with clinical improvement lasting for weeks.
After this, symptoms due to either esophageal or gastric stricture begin to form. Most adults who ingest liquid alkali will
develop severe esophageal and usually gastric injury that results in stricture formation. Children, with usually more limited
exposure from accidental ingestions, are less likely to have severe injuries.

80.Which of the following statement(s) is/are true concerning nonresectional therapy for esophageal carcinoma?

a. Radiation therapy can be associated with five-year survival rates equal to surgery
b. Esophageal intubation to provide palliation for esophageal cancer is associated with minimal morbidity and
mortality
Asir Surgery MCQs Bank. © 1422H-2002- first impression ©
185
This project was raised after an idia by Dr. Gharama Al-Shehri (consultant surgeon). Developed and typed by Dr. Ghazi Al-Shumrani (intern).
Esophagus, stomach & duodenum
c. Endoscopic laser fulguration is successful in up to 75% of patients
d. There is little or no role for surgical bypass for unresectable esophageal carcinoma
Answer: c, d

Therapy of esophageal carcinoma is influenced by the knowledge that in most of these patients, local tumor invasion or
distant metastatic disease preclude cure. While squamous cell carcinoma is generally regarded as a radiosensitive and
therefore potentially curable tumor, radiation therapy has not achieved cure in most patients. Although “curative” super
voltage radiation techniques have been employed, the average five-year survival after such treatment is between 6 and 10%
in most series. This is somewhat poorer than five-year survival rates after resection which usually range between 10 and
15%. A variety of endoesophageal tubes have been used for palliation in patients with esophageal carcinoma. Basically,
these tubes are divided into two types: the pulsion tubes, which are pushed through the tumor with the aid of an
esophagoscope; and the traction or pull-through tubes, which are pulled into place by downward traction through a
gastrostomy. As in the case with many conceptually simple procedures, implementation in the clinical setting is problematic.
Transoral esophageal intubation is associated with an overall mortality of 14% and a complication rate of 25%, the latter
due to perforation of the esophagus, migration of the tubes, or obstruction of the tubes by food or tumor overgrowth. More
recently, a variety of expandable intraesophageal metallic stents have been used to achieve palliation in patients with
unresectable esophageal carcinoma. Additional expertise with this technique is being acquired through a multi-institutional
trial currently underway in the United States. Endoscopic laser fulguration of esophageal carcinoma has been used to
achieve temporary relief of the esophageal obstruction in patients with unresectable tumors. Generally, multiple sessions are
required to resect sufficient tumor to achieve adequate lumen, but functional success with restoration of a comfortable
volume can be achieved in 75–80%. Although a variety of surgical procedures such as substernal gastric or colon bypasses
have been developed as palliative internal bypasses of unresectable esophageal carcinoma, the limited survival in these
unresectable patients can be high and the mortality rates, between 15 and 25%, do not currently justify their use.

81.Benign tumors of the esophagus are rare constituting less than 1% of esophageal neoplasms. Which of the following
statement(s) is/are true concerning benign esophageal neoplasms?

a. Most esophageal polyps are located just above the gastroesophageal junction
b. Malignant degeneration of leiomyomas of the esophagus is a frequent occurrence
c. An asymptomatic leiomyoma can be safely observed and followed with periodic barium esophograms and
endoscopic ultrasonography
d. Most leiomyomas of the esophagus require esophagectomy
Answer: c

Leiomyomas represent the most common benign intramural esophageal tumor and characteristically occur in patients
between 20 and 50 years of age. More than 80% of esophageal leiomyomas occur in the middle and lower thirds of the
esophagus. Tumors less than 5 cm in size rarely cause symptoms. When larger than this, dysphagia, retrosternal pressure
and pain are the common complaints. Bleeding more often occurs with the malignant form of the tumor, leiomyosarcoma.
Malignant degeneration of leiomyomas is exceedingly rare. An asymptomatic leiomyoma or one discovered incidentally on
a barium swallow examination can be safely observed and followed with periodic barium esophograms and endoscopic
ultrasonography. Although excision of the esophageal mass provides the only definitive tissue diagnosis, the characteristic
radiographic appearance, slow growth rate, and low risk of malignant degeneration and the ability to follow leiomyomas
with endoscopic ultrasonography justify conservative management. Tumors that are symptomatic or larger than 5 cm in
diameter should be excised. Tumors of the middle third of the esophagus are approached through a right thoracotomy, while
those in the distal third are approached through a left thoracotomy. Once the esophagus is encircled and the tumor located,
the overlying longitudinal muscle is split in the direction of its fibers. The tumor is then gently dissected away from the
contiguous underlying submucosa and adjacent muscle. When enucleation of the tumor is complete, the longitudinal
esophageal muscle is reapproximated, although a large extramucosal defect may be left without complication. Giant
leiomyomas of the cardia and adjacent stomach may require esophageal resection for their removal. When resection is
complete, leiomyomas virtually never recur.
Benign polyps of the esophagus are rare and typically arise in the cervical esophagus. Most are seen in older men and are
frequently attached to the cricoid cartilage. Histologically, they are composed of fibrovascular tissue with varying amounts
of associated fat.

82.In an effort to improve survival following esophageal resection, trials of multi-modality therapy in combination with
surgery have been completed. Which of the following statement(s) is/are true concerning such treatment?
Asir Surgery MCQs Bank. © 1422H-2002- first impression ©
186
This project was raised after an idia by Dr. Gharama Al-Shehri (consultant surgeon). Developed and typed by Dr. Ghazi Al-Shumrani (intern).
Esophagus, stomach & duodenum

a. Therapy appears to be indicated in squamous cell carcinoma but not adenocarcinoma


b. No residual carcinoma may be found in the resected specimen in up to 20% of patients
c. Nonrandomized trials would suggest improved survival compared to patients receiving surgery alone
d. Perioperative morbidity is increased due to preoperative radiation and chemotherapy
Answer: b, c,

Combined preoperative chemotherapy and radiation therapy before transhiatal esophagectomy for carcinoma has provided
encouraging survival statistics. The treatment consists of preoperative chemotherapy with three weeks of cisplatin,
vinblastine, and 5-fluorouracil, concurrent with 3750 to 4500 cGy of radiation therapy. Although hematologic toxicity and
radiation esophagitis are common and preoperative deaths due to bone marrow suppression can occur, there is no increase in
perioperative morbidity when compared to patients with no preoperative therapy. In one study, 24% of patients had no
residual carcinoma in the resected specimen (T0, N0 status). At mean follow-up of 36 months, the mean survival in this
series was 29 months, a clear improvement over the 12-month median survival time with transhiatal esophagectomy alone
in historical controls.

83.Which of the following statement(s) is/are correct concerning the management of a patient with a caustic esophageal
or gastric injury?

a. Corticosteroids should be administered immediately


b. Complete endoscopic examination of the esophagus and stomach should be completed
c. Patients requiring operative intervention are best explored through the abdomen
d. If organ resection is indicated, restoration of alimentary continuity should be deferred until the patient has
recovered from the acute insult
e. In patients with esophageal stricture following second and third degree burns, dilatation therapy should be
instituted as soon as possible after the injury
Answer: b, c, d

Acute caustic ingestion is indication for hospitalization. Initial management centers on stabilizing the patient and assessing
the severity of injury. Oral intake should be withheld and hypovolemia corrected with intravenous fluids. Careful
observation for evidence of airway obstruction is mandatory. Broad spectrum antibiotics are indicated once the diagnosis of
substantial esophageal injury has been established to diminish the risk of pulmonary infection from aspiration as well as
bacterial invasion through the damaged esophageal wall. Although corticosteroids have been advocated in the acute phase of
caustic ingestion to minimize subsequent stricture formation, their efficacy has not been established. Furthermore, because
steroids may mask signs of sepsis, visceral perforation, and impair healing, their use in caustic esophageal injury is
potentially deleterious and therefore is not recommended.
A contrast esophogram is the best way to make the diagnosis of esophageal perforation and should be performed if the
diagnosis is suspected at the time of admission or in subsequent followup. Esophagoscopy should be performed soon after
admission to establish whether significant esophageal injury has occurred and to permit grading of the severity of injury.
Although in the past it was taught that the endoscope should not be advanced beyond the first burned area, more recently
complete examination of the esophagus and stomach has been recommended, especially if severe burns are not detected
proximally. The use of a pediatric endoscope and adequate sedation can allow this procedure to be accomplished safely.
Patients with caustic liquid ingestion that necessitate operative intervention are generally best explored through the
abdomen. This approach permits assessment of the injury to the intraabdominal organs as well as resection of areas of full
thickness gastric necrosis. Although only the lower esophagus is well visualized through the diaphragmatic hiatus, if an
esophageal resection is required, transhiatal esophagectomy without thoracotomy is readily performed by the addition of a
cervical incision. When esophageal gastric resection for acute caustic injury is required, restoration of alimentary continuity
should be deferred until the patient has recovered from the acute insult and the development of chronic stricture formation
and retained organs can be evaluated. Esophageal stricture formation following second and third degree burns is the rule,
and dilatation therapy has been the traditional therapy for chronic caustic esophageal strictures. It is important that dilatation
not be instituted until at least six to eight weeks after the injury, when reepithelialization is complete, in order to minimize
the risk of perforation.

84.In regard to the arterial blood supply to the stomach, which of the following statement(s) is/are true?

a.The right gastric artery, a branch of the superior mesenteric artery, supplies the gastric antrum
Asir Surgery MCQs Bank. © 1422H-2002- first impression ©
187
This project was raised after an idia by Dr. Gharama Al-Shehri (consultant surgeon). Developed and typed by Dr. Ghazi Al-Shumrani (intern).
Esophagus, stomach & duodenum
b.Because of rich intramural collaterals, gastric viability may be preserved after ligation of all but one major artery
c.In cases of celiac artery occlusion, gastric viability is maintained collaterally via pancreaticoduodenal arcades
d.The left gastroepiploic artery is a branch of the celiac trunk
Answer: b, c

The stomach is an extremely well-vascularized organ, supplied by 5 major arterial distributions and protected from ischemia
by rich intramural and extramural collaterals. The left gastric artery and right gastric artery, derived from the celiac
distribution, supply the lesser curvature of the stomach. The right gastroepiploic artery, derived from the gastroduodenal
artery, and the left gastroepiploic artery, from the splenic artery, traverse the greater curvature. The area adjacent to the
spleen receives multiple short gastric arterial branches. In instances of celiac arterial occlusion, the superior mesenteric
artery supplies the stomach collaterally via the pancreaticoduodenal arcades which connect with the gastroduodenal artery.
The stomach may be widely mobilized for use in reconstructive procedures, for example, during trans-hiatal
esophagectomy. Advantage is taken of the abundant blood supply and collaterals of the stomach during mobilization; gastric
viability is usually preserved if one major arterial supply is preserved.

85.At a cellular level, the major stimulant(s) of acid secretion by the gastric parietal cell is/are:

a.Histamine
b.Prostaglandin E2
c.Acetylcholine
d.Gastrin
e.Norepinephrine
Answer: a, c, d

The three major stimulants of acid secretion by the parietal cell are acetylcholine, gastrin, and histamine. Acetylcholine is
released from cholinergic nerve endings in close proximity to parietal cells and binds to muscarinic receptors. Cholinergic
stimulation of parietal cells is coupled to hydrolysis of membrane-associated lipids (termed phosphatidylinositides) and
leads to increases in intracellular calcium. Histamine is released from mast cells in the lamina propria and reaches parietal
cells by diffusion. Histamine occupies H2 receptors that may be selectively blocked by agents such as cimetidine. Histamine
stimulation of parietal cell acid secretion is mediated by a cyclic AMP-dependent pathway. Gastrin is delivered to the fundic
mucosa by the systemic circulation from its source in the antrum and duodenum. Like acetylcholine, gastrin causes
increases in membrane phosphoinositol turnover and increases intracellular calcium.
Activation of parietal cells by acetylcholine, gastrin or histamine can be blocked by somatostatin. Local release of
somatostatin is physiologically important in modulating postprandial gastric acid secretion. Prostaglandin E2 and its
synthetic derivatives are potent inhibitors of histamine-stimulated acid secretion.

86.Which of the following statement(s) regarding the vagus nerves is/are true?

a.The right and left vagus nerves derive from a nerve plexus inferior to the tracheal bifurcation
b.The posterior vagus nerve is closely applied to the intrathoracic esophagus
c.The anterior vagus supplies a hepatic division which passes to the right in the lesser omentum
d.Approximately 90% of vagal fibers are afferent, transmitting information from the gastrointestinal tract to the central
nervous system
e.The vagus nerves transmit gastroduodenal pain sensations associated with peptic ulceration
Answer: a, c, d

The left and right vagus nerves are formed from a periesophageal nerve plexus between the tracheal bifurcation and the
diaphragm. As they pass through the esophageal hiatus, the anterior vagus is closely applied to the esophagus; the posterior
vagus lies intermediate in position between the esophagus and the aorta. The anterior vagus supplies the hepatic division
which provides parasympathetic innervation to the liver and biliary tract. The hepatic division is usually easily seen in the
thin gastrohepatic omentum and is constant in location. The hepatic division is a useful anatomic landmark in vagotomy
procedures. The posterior vagus nerve supplies fibers to the celiac division. After giving off hepatic and celiac divisions,
both anterior and posterior vagus nerves supply branches to the gastric wall. Surprisingly, only 10% of vagal fibers are
efferent, secretomotor fibers; almost 90% are afferent. Sensations of gastric pain are carried in sympathetic fibers, and
vagotomy does not alter perception of painful gastric conditions or stimuli.

Asir Surgery MCQs Bank. © 1422H-2002- first impression ©


188
This project was raised after an idia by Dr. Gharama Al-Shehri (consultant surgeon). Developed and typed by Dr. Ghazi Al-Shumrani (intern).
Esophagus, stomach & duodenum

87.Important stimulants of gastrin release from endocrine cells in the antrum include:

a.Acidification of the antral lumen


b.Small peptide fragments and amino acids from luminal proteolysis
c.Locally released somatostatin
d.Dietary fats
Answer: b

Gastrin is processed to 34-and 17-amino acid forms in endocrine cells in the gastric antrum. In addition to well-recognized
stimulatory actions on gastric acid secretion, gastrin promotes mucosal growth of the gastric fundus and small intestine. The
most important stimulant of gastrin release is a meal. Small peptide fragments and amino acids that result from intragastric
proteolysis are the food components that stimulate gastrin release. Ingested fats and carbohydrates have no significant
effect. In this regard, intraluminal pH strongly affects gastrin secretion. If intragastric pH is maintained above 3 after
ingestion of a meal, gastrin release is strongly potentiated. Pernicious anemia and atrophic gastritis, which produce chronic
achlorhydria, are associated with fasting hypergastrinemia and an exaggerated gastrin meal response. Conversely, antral
acidification strongly inhibits gastrin secretion. Locally-released somatostatin mediates the effects of luminal acidification,
inhibiting gastrin secretion.

88.Which of the following statements regarding human gastric acid secretion is/are true?

a.Fasting acid secretion, normally 2 to 5 mEq/h, is due to ambient vagal tone and histamine secretion
b.Truncal vagotomy decreases basal secretion by 80%
c.Histamine2 receptor antagonist administration can decrease basal acid secretion by 80%
d.Fasting acid secretion, normally 5 to 10 mEq/h, is due to circulating levels of gastrin
Answer: a, b, c

Both vagal tone and locally secreted histamine are presumed to be the determinants of basal acid secretion in humans.
Gastrin does not have a role in basal acid secretion in normal individuals.
Parietal cell activation and the resultant acid secretory response is greater to a combination of agonists than the sum of the
responses to the agents used singly. This increase in responsiveness is termed potentiation. Potentiating interactions are most
apparent when the stimulants use different second messenger systems, for example, acetylcholine and histamine.
Conversely, blockade of receptors to one stimulant also blocks responsiveness to the other agonist. Because of this
interaction, blockade of histamine receptors by agents like cimetidine decreases responsiveness to acetylcholine. Blockage
of acetylcholine release by vagotomy decreases responsiveness to histamine secreted by gastric mast cells.
Both vagotomy and histamine2 receptor antagonists decrease basal acid secretion by approximately 80%.

89.As a meal is emptied from the stomach, gastric acid secretion gradually returns to baseline. Which of the following
statements correctly characterize control of gastric acid secretion?

a.In humans, the most important inhibitory influence on gastrin release is exposure of the gastric mucosa to luminal
acid
b.Acidification of the antral lumen causes reciprocal increases in somatostatin release and decreases in gastrin secretion
c.Antral distension stimulates gastric acid secretion
d.Acidification of the duodenal bulb inhibits gastric acid secretion
e.Exposure of the duodenum to hyperosmolar solutions inhibits acid secretion
Answer: a, b, d, e

Inhibitory regulation of gastric acid secretion is accomplished by central nervous system, gastric and small intestinal
mechanisms. In humans, the most clearly established gastric inhibitory influence is suppression of gastrin release by
exposure of the antral mucosa to luminal acid. Antral acidification causes release of gastric mucosal somatostatin which is
linked reciprocally to decreases in gastrin secretion. Antral distension inhibits gastric acid secretion.
The inhibitory phase of gastric acid secretion begins with entry of the products of digestion into the proximal duodenum.
Acidification of the duodenal bulb and exposure of the duodenum to hyperosmolar solutions and those containing fat
potently inhibit acid secretion.

Asir Surgery MCQs Bank. © 1422H-2002- first impression ©


189
This project was raised after an idia by Dr. Gharama Al-Shehri (consultant surgeon). Developed and typed by Dr. Ghazi Al-Shumrani (intern).
Esophagus, stomach & duodenum

90.Which of the following statements correctly characterizes gastric motor activity associated with ingestion of a meal?

a.Ingested gastric volumes are accommodated with little increase in pressure by reflex relaxation of the proximal
stomach
b.Receptive gastric accommodation is unaffected by proximal gastric vagotomy
c.In humans, liquid emptying occurs more quickly than solid emptying
d.Gastric emptying of liquids is not affected by proximal gastric vagotomy
Answer: a, c

With ingestion of a meal, increasing gastric volumes are accommodated with little increase in intragastric pressure by
relaxation of the proximal stomach. This process, termed receptive relaxation, is mediated by a reflex carried by the vagal
nerve. After the meal has been ingested, the proximal stomach is the predominant determinant of the rate of gastric
emptying of liquids due to the gastroduodenal pressure gradient generated by proximal gastric contractions. Liquid
emptying occurs more rapidly than emptying of solids, in part, because liquids are not subject to the sieving actions of the
pylorus.
Truncal and proximal gastric vagotomy abolish receptive relaxation. Following vagotomy, an increased gastroduodenal
pressure gradient is observed and correlates with accelerated liquid emptying. Emptying of solids is usually not significantly
altered by proximal gastric vagotomy.

91.It is widely agreed that the gastric mucosa secretes bicarbonate in addition to acid. Gastric secretion of bicarbonate
is correctly characterized by which of the following statements?

a.Bicarbonate is secreted by chief cells within gastric crypts


b.Gastric bicarbonate secretion is stimulated by acetylcholine
c.Gastric bicarbonate secretion during fasting results in luminal pH above 6 in normal individuals
d.Prostaglandin E2 is a potent stimulant of gastric bicarbonate secretion
Answer: b, d

The gastric cells responsible for bicarbonate secretion are believed to be surface mucous cells facing the gastric lumen
between crypts. Although the total amount of gastric bicarbonate secreted is only a small fraction of total acid secretion, pH
close to neutrality is maintained near the mucosal surface while bulk luminal pH is highly acidic. Cholinergic agonists,
vagal stimulation, and sham feeding all increase gastric bicarbonate secretion. Prostaglandin E2 and its synthetic derivatives
are potent stimulants of bicarbonate secretion as well. Conversely, indomethacin and other drugs that inhibit prostaglandin
formation decrease mucosal bicarbonate secretion.

92.Gastric mucosal blood flow is regulated by neural, hormonal, and locally active influences. Which of the following
statements correctly characterizes gastric blood flow?

a.Stimulation of sympathetic nerves supplying the stomach is followed by gastric mucosal hyperemia and increased
total gastric blood flow
b.Vagal nerve stimulation is accompanied by decreased gastric mucosal blood flow
c.Stimulants that increase acid secretion increase mucosal blood flow
d.In humans, prostaglandins increase mucosal blood flow at doses that inhibit gastric acid secretion
Answer: c, d

Because the gastric mucosa is metabolically highly active, control of gastric mucosal blood flow is of great physiologic
significance. Almost all stimuli that increase acid secretion also increase gastric blood flow. A large number of
gastrointestinal hormones stimulate gastric blood flow, most because of their ability to increase acid secretion. Thus, gastrin
is a potent stimulant of blood flow, in proportion to its ability to increase acid secretion. Vagal nerve stimulation has the net
effect of increasing mucosal and total gastric blood flow; sympathetic nerve stimulation is accompanied by opposite effects.
Prostaglandins are important endogenous regulators of gastric blood flow. Prostaglandins of the E class increase blood flow
at doses that suppress acid secretion. Inhibition of cyclo-oxygenase activity by indomethacin causes a reduction in resting
gastric blood flow.

Asir Surgery MCQs Bank. © 1422H-2002- first impression ©


190
This project was raised after an idia by Dr. Gharama Al-Shehri (consultant surgeon). Developed and typed by Dr. Ghazi Al-Shumrani (intern).
Esophagus, stomach & duodenum
93.Which of the following statements regarding intrinsic factor is/are correct?

a.Intrinsic factor is produced in chief cells located in the gastric fundus


b.Total gastrectomy is followed by folate deficiency due to vitamin malabsorption secondary to intrinsic factor
deficiency
c.Intrinsic factor secretion, like that of acid, is stimulated by gastrin, histamine, and acetylcholine
d.Intrinsic factor deficiency accompanies H pylori-caused antral gastritis
Answer: c

The gastric mucosa is the site of production of intrinsic factor, which is a necessary co-factor for the absorption of vitamin
B12 by the ileal mucosa. Total gastrectomy and atrophic gastritis involving the proximal oxyntic mucosa are regularly
followed by vitamin B12 deficiency, manifest as pernicious anemia. Acid-secreting parietal cells are the site of intrinsic
factor synthesis. Like acid secretion, intrinsic factor secretion is stimulated by gastrin, histamine, and acetylcholine

94.A 24-year-old woman develops epigastric pain and has a diagnosis of duodenal ulcer confirmed by
esophagogastroduodenoscopy. The patient is in the third month of a pregnancy. The most appropriate treatment would
be:

a.Proximal gastric vagotomy


b.Misoprostol 400 mg b.i.d.
c.Sucralfate 1 gm q.i.d.
d.Cimetidine 400 mg b.i.d.
Answer: c

Cimetidine, ranitidine, famotidine and newer H2 receptor antagonists bind competitively to parietal cell histamine receptors
to produce reversible inhibition of acid secretion. While the pharmacokinetic profiles of the H2 receptor antagonists differ,
when administered at equipotent doses, the agents produce similar degrees of acid suppression and similar rates of ulcer
healing. Cimetidine interacts with the hepatic microsomal enzyme system and may increase blood levels and
pharmacological effects of drugs that depend upon hepatic metabolism.
Misoprostol, a derivative of prostaglandin E, causes endoscopic healing in 60% of patients at 4 weeks. The major side effect
of misoprostol is diarrhea due to effects upon gut smooth muscle contractility. Uterine bleeding has been reported in some
women using the drug and the agent has potential abortifacient actions. For this reason, misoprostol is contraindicated in
pregnancy.
Sucralfate, the aluminum salt of sulfated sucrose, has virtually no systemic absorption, and for this reason, is the drug of
choice for pregnant patients. When administered at a dose of 1 gm four times daily, 80% of ulcers will heal by 6 weeks.

95.Helicoabacter pylori has been investigated as a possible etiologic agent in duodenal ulceration. Which of the
following statement(s) regarding H pylori infection in humans is/are correct?

a.H pylori may be isolated from antral gastric mucosa in nearly 100% of patients with active duodenal ulceration but
only 1–2% of normal volunteers
b.H pylori possess cell surface receptors that bind to small intestinal mucous cells
c.Therapeutic regimens for duodenal ulcer that eliminate the organism are associated with lower ulcer recurrence rates
than those in which the organism persists
d.The incidence of the organism in the normal population increases with age
e.Antral gastritis is associated with development of duodenal ulcer
Answer: c, d, e

Helicobacter pylori has received enormous investigative attention in recent years as a possible infectious cause of peptic
ulceration. The evidence that H pylori causes ulcers is substantial but largely inferential. Antral gastritis is nearly always
present in patients with duodenal ulceration. H pylori infestation of antral mucosa is believed to cause gastritis. While
normal small intestinal cells do not permit H pylori binding, areas of gastric metaplasia are usually demonstrated in the
duodenal mucosa immediately surrounding the ulcer. Resolution of gastritis follows eradication of the organism and drug
regimens that are bactericidal are associated with lower rates of ulcer recurrence than those that have no anti-bacterial
actions.

Asir Surgery MCQs Bank. © 1422H-2002- first impression ©


191
This project was raised after an idia by Dr. Gharama Al-Shehri (consultant surgeon). Developed and typed by Dr. Ghazi Al-Shumrani (intern).
Esophagus, stomach & duodenum
However, it is clear that not all patients infested with H pylori develop ulceration. Half of patients with dyspepsia but no
ulceration have evidence of H pylori infestation and 20% of healthy volunteers can be demonstrated to harbor the organism.
The incidence of infestation increases with age in the asymptomatic population.

96.A 40-year-old male undergoes treatment of acute duodenal ulceration with cimetidine 400 mg b.i.d. and has
resolution of symptoms by 6 weeks. The medication is continued as a nocturnal maintenance dose at the end of a three
month treatment course. Recurrent symptoms develop 6 months after initial diagnosis and repeated endoscopic
examination reveals recurrent ulceration. Biopsies of antral mucosa demonstrate moderate gastritis and the presence of
H pylori. Medical management designed to eradicate H pylori and heal ulceration should include which of the
following agents?

a.Cimetidine
b.Bismuth subcitrate
c.Amoxicillin
d.Metronidazole
e.Vancomycin
Answer: a, b, c, d

The observation that H pylori infection has an important role in ulcer pathogenesis has led to development of antimicrobial
therapy for ulceration. Most successful regimens are based on a bismuth compound (colloidal bismuth subsalicylate or
colloidal bismuth subcitrate) plus metronidazole, alone or in combination with amoxicillin or tetracycline. Bismuth
compounds act locally and achieve gastric concentrations above the MIC for 90% of H pylori isolates. Metronidazole is
secreted into the stomach at high concentration, and the in vivo activity of metronidazole is not diminished by gastric
acidity. Triple therapy with bismuth, metronidazole, and tetracycline or amoxicillin eradicates H pylori in 90% of cases,
compared to 0% eradication with ranitidine. Inclusion of an H2 receptor antagonist or omeprazole has been reported to
increase efficacy of antimicrobial therapy. Currently, antimicrobial therapy has been recommended for peptic ulcer disease
resistant to conventional therapy, including patients with ulcer relapse while on maintenance therapy and failure to heal in
spite of H2 receptor antagonist or omeprazole therapy.

97.Development of duodenal ulceration is dependent upon gastric acid secretion. Which of the following statements
correctly characterizes acid secretion in duodenal ulcer patients?

a.Groups of duodenal ulcer patients demonstrate decreased basal acid secretion


b.Maximal acid output to histamine averages 40 mEq/h in duodenal ulcer patients, twice that of normal
c.Tissue gastrin levels, on average, are twice normal in patients with active ulceration
d.Exogenously administered somatostatin is ineffective in suppressing acid secretion in patients with active ulceration
Answer: b

The formation of duodenal ulcer is dependent on gastric secretion of acid and pepsin. As a group, duodenal ulcer patients
have an increased capacity for gastric acid secretion relative to normal individuals, manifest by increased basal acid
secretion, increased acid response to meal ingestion, and increased responsiveness to histamine stimulation. No definite
evidence links abnormalities in gastrointestinal hormone secretion with increased acid secretion. Tissue gastrin content and
circulating levels of gastrin are normal (excluding Zollinger-Ellison syndrome patients). Secretion of endogenous
somatostatin and responsiveness to exogenously administered somatostatin are likewise normal.

98.A 45-year-old man undergoes proximal gastric vagotomy for treatment of intractable duodenal ulceration. What
physiologic alterations might be anticipated as a consequence of the operation?

a.Reduction of basal acid secretion by approximately 25%


b.Accelerated gastric emptying of liquids
c.Accelerated gastric emptying of solids
d.Fasting hypergastrinemia
e.Postprandial hyperinsulinemia
Answer: b, d

Asir Surgery MCQs Bank. © 1422H-2002- first impression ©


192
This project was raised after an idia by Dr. Gharama Al-Shehri (consultant surgeon). Developed and typed by Dr. Ghazi Al-Shumrani (intern).
Esophagus, stomach & duodenum
Division of cholinergic vagal fibers directly affects parietal cell acid secretion by reducing stimulatory input. Basal acid
secretion is diminished by approximately 80% and maximal acid output in response to pentagastrin stimulation is reduced
by about 70%. Fasting hypergastrinemia and an exaggerated gastrin response to meal ingestion is observed due to loss of
feedback inhibition of gastrin release and gastrin cell hyperplasia. Release of pancreatic polypeptide, secretin and
cholecystokinin may be decreased.
Proximal gastric vagotomy accelerates gastric emptying of liquids due to a loss of receptive relaxation. In contrast, gastric
emptying of solids is usually not affected by proximal gastric vagotomy.

99.Which of the following statements regarding postoperative rates of recurrent ulcer and dumping is/are correct?

a.Truncal vagotomy and antrectomy is associated with persistent dumping in 10–15% of patients
b.Recurrent ulceration following truncal vagotomy and pyloroplasty is observed in 25% of patients within 10 years of
operation
c.Patients that undergo proximal gastric vagotomy have a risk of recurrent ulcer of 10–15% and a risk of persistent
dumping approximating 1%
d.Recurrent ulceration occurs in 5% of patients that undergo truncal vagotomy and antrectomy
Answer: a, c

Surgical recommendations for treatment of peptic ulceration should be based upon safety, freedom from long-term
postoperative symptoms, and avoidance of recurrent ulceration. Proximal gastric vagotomy has an operative mortality of
less than 1% and a risk of persistent dumping symptoms of approximately 1%. The low incidence of postoperative
symptoms is associated with a relatively high risk of recurrent ulceration, estimated to be 10% to 15% at 5 years
postoperatively. After truncal vagotomy and pyloroplasty, dumping is initially present in 10%, and is persistent or severe in
1%. Recurrent ulceration is observed in 10% of patients that undergo truncal vagotomy and pyloroplasty. Truncal vagotomy
and antrectomy is associated with the lowest risk of recurrent ulceration, 1–2%, but the greatest incidence of postoperative
dumping symptoms, 10–15%.

100.Which of the following statement(s) is/are correct with regard to pyloric obstruction secondary to peptic
ulceration?

a.Pyloric obstruction is suggested by hypochloremic hyponatremic alkalosis


b.Pyloric obstruction is suggested by hypochloremic hypokalemic alkalosis
c.Approximately 80% of patients with benign gastric outlet obstruction obtain permanent relief of symptoms by
endoscopically-directed balloon dilatation
d.The lifetime risk of pyloric obstruction in peptic ulcer patients is 40%
Answer: b

Repeated episodes of ulceration and healing can lead to scarring and pyloric stenosis. The lifetime risk of this complication
approximates 10%. Gastric outlet obstruction is characterized by the development of hypochloremic hypokalemic alkalosis
due to loss of HCl through vomiting and renal compensatory mechanisms that conserve H+ at the expense of secreted K+.
Although 85% of pyloric stenoses are technically amenable to balloon dilatation, fewer than 1 in 3 will achieve permanent
relief of symptoms through this means.

101.A 42-year-old man with a recently diagnosed duodenal ulcer develops melena and near-syncope. After fluid
resuscitation, upper gastrointestinal endoscopy is performed. During the examination, a 1 cm ulcer is noted in the
proximal duodenum. A fresh clot is observed within the ulcer and blood is noted to be oozing around the clot. Optimal
therapy would consist of which of the following?

a.Angiographic embolization of the gastroduodenal artery


b.Irrigation of the clot followed by endoscopic application of a heat probe
c.Transfusion and intravenous cimetidine
d.Angiographic infusion of vasopressin into the gastroduodenal artery
e.Transfusion and oral omeprazole
Answer: b

Asir Surgery MCQs Bank. © 1422H-2002- first impression ©


193
This project was raised after an idia by Dr. Gharama Al-Shehri (consultant surgeon). Developed and typed by Dr. Ghazi Al-Shumrani (intern).
Esophagus, stomach & duodenum
The ability to visualize bleeding duodenal ulcers endoscopically has led to attempts to treat hemorrhage endoscopically.
Thermal coagulation may be achieved by bipolar electrocoagulation or by direct application of heat through a probe. An
NIH Consensus Development Conference has recommended endoscopic hemostatic therapy in selected patients.
Hematemesis, age over 60, and serious medical co-morbidity are clinical features that mandate endoscopic therapy.
Rebleeding during hospitalization and the endoscopic findings of visible vessel, oozing, or bleeding associated with an
adherent clot are other indications for endoscopic hemostasis. Operative intervention is appropriate for massive hemorrhage
leading to shock or cardiovascular instability, prolonged blood loss requiring continuing transfusion, recurrent bleeding
during medical therapy or after endoscopic therapy, and recurrent hemorrhage requiring hospitalization. Operative therapy
should consist of duodenotomy with direct ligation of the bleeding vessel within the ulcer base followed by a procedure to
permanently reduce acid production.

102.A 50-year-old patient has undergone truncal vagotomy and antrectomy with Billroth II reconstruction two years
ago. The patient now complains of recurrent postprandial pain, nausea, and vomiting. Endoscopic examination reveals
bile in the stomach; endoscopic biopsies demonstrate histologic evidence of moderately severe gastritis. No other
endoscopic abnormalities are noted. Appropriate therapy could include:

a.Octreotide administration
b.Total gastrectomy
c.Conversion of Billroth II gastrojejunostomy to Billroth I gastroduodenostomy
d.Conversion of Billroth II gastrojejunostomy to Roux-en-Y gastrojejunostomy
e.Roux-en-Y hepaticojejunostomy
Answer: d

Symptoms related bile reflux gastritis occur transiently in 10% to 20% of patients after truncal vagotomy and resection or
drainage. Symptoms persist in only 1% to 2%. No completely satisfactory solution to bile reflux gastritis exists. Medicinal
and dietary treatments have not been proven to be beneficial. Operative diversion of biliary secretions away from the gastric
mucosa by construction of a Roux-en-Y gastrojejunostomy with an intestinal limb of 50 to 60 cm has been widely reported.
The procedure eliminates bilious vomiting in nearly 100% of patients, but pain persists in up to 30%, and 20% develop
delayed gastric emptying as a result of the procedure.

103.A 50-year-old male with a 2 year history of duodenal ulceration develops sudden, severe epigastric pain 4 hours
prior to evaluation. Physical examination reveals T 101° F, pulse 80, BP 125/90, diminished bowel sounds, and
abdominal muscular rigidity. An upright chest x-ray reveals pneumoperitoneum. At laparotomy, an anterior perforation
in the first portion of the duodenum is observed. Optimal treatment would include:

a.Omental patch of the perforation followed by truncal vagotomy and antrectomy after 8 weeks
b.Omental patch of the perforation followed by truncal vagotomy and pyloroplasty after 8 weeks
c.Omental patch of the perforation followed by chronic cimetidine administration
d.Omental patch of the perforation plus proximal gastric vagotomy
e.Omental patch of the perforation only
Answer: d

Simple omental patching of a perforation in patients with chronic ulcer disease does not yield satisfactory long-term results.
Up to 80% of patients so treated develop recurrent ulceration and 10% develop secondary complications. A definitive ulcer
operation should be performed during the initial laparotomy if the following circumstances apply: 1) there has been no
preoperative shock, 2) the perforation has been present for less than 48 hours, and 3) no life-threatening medical co-
morbidity exists. Omental patching of the perforation combined with proximal gastric vagotomy is a preferred approach
because it combines safety, freedom from disabling postoperative symptoms, and a low rate of recurrent ulceration.

104.Which of the following clinical circumstances have been identified as predisposing factors for the development of
stress ulceration?

a.Intraperitoneal sepsis
b.Hemorrhagic shock

Asir Surgery MCQs Bank. © 1422H-2002- first impression ©


194
This project was raised after an idia by Dr. Gharama Al-Shehri (consultant surgeon). Developed and typed by Dr. Ghazi Al-Shumrani (intern).
Esophagus, stomach & duodenum
c.Isolated tibial fracture
d.50% total surface area second degree burn
e.Adult respiratory distress syndrome
Answer: a, b, d, e

Several risk factors or predisposing clinical conditions have been identified for stress ulceration. Specific risk factors
include adult respiratory distress syndrome, multiple trauma, major burn of over 35% of body surface area, oliguric renal
failure, large transfusion requirements, hepatic dysfunction, hypotension, prolonged surgical procedures, and sepsis from
any source. A direct correlation has been shown between acute upper gastrointestinal hemorrhage and the severity of critical
illness.

105.Type I gastric ulcers are located in the gastric body, usually along the lesser curvature. Which of the following
statements correctly characterize type I gastric ulcers?

a.Normal to low acid secretion


b.Associated duodenal ulceration
c.High frequency of blood group A
d.Associated hypergastrinemia frequent
Answer: a, c

Gastric ulcers are divided into categories based on their location and gastric acid secretory status. A type I gastric ulcer is an
ulcer in the body of the stomach, usually along the lesser curvature, associated with large volumes of secretion with a low to
normal acid output. Type I ulcers are not associated with duodenal, pyloric, or prepyloric mucosal abnormalities. There is a
slight predominance of patients with blood group A in this type of gastric ulcer.
Type II gastric ulcer is located in the body of the stomach in combination with a duodenal ulcer. These patients are usually
acid hypersecretors. About 23% to 25% of gastric ulcers are type II. A type III gastric ulcer is characterized as a prepyloric
ulcer and accounts for about 23% of lesions. Patients with this lesion are typically acid hypersecretors.
Type IV gastric ulcer occur high on the lesser curvature near the gastroesophageal junction. In the United States, the
incidence of type IV gastric ulcer is less than 10%.

106.Which of the following statement(s) is/are correct regarding gastric ulcers greater than 3 cm in size?

a.Giant gastric ulcers occur in 30–40% of cases along the greater curvature
b.The risk of malignancy increases with increasing size of the ulcer
c.The treatment of choice for giant gastric ulcer is resection to include the ulcer
d.Giant gastric ulcer is a complication of intraarterial hepatic chemotherapy
Answer: b, c

A giant gastric ulcer is defined as an ulcer whose diameter is 3 cm or greater. The lesser curvature is the most common site,
with only 3% to 4% occurring along the greater curvature. Gastric ulcers often penetrate into contiguous structures such as
spleen, pancreas, liver, and transverse colon, and can be falsely diagnosed as a nonresectable malignancy, despite normal
biopsy results. Most reports cite an incidence of malignancy ranging from 6% to 30%, increasing with the size of the ulcer.
Because of the high likelihood of complications from giant gastric ulcer, early operation is the treatment of choice. The
operation of choice is resection, including the ulcer. If the ulcer has penetrated adjacent structures and cannot be dissected
free, the stomach wall can be incised circumferentially, leaving the ulcer intact and behind, and the gastric resection
completed.

107.With regard to benign gastric ulceration, the most common location of disease is which of the following?

a.Along the greater curvature


b.Immediately distal to the esophagogastric junction along the lesser curvature
c.In the area of the incisura angularis along the lesser curvature
d.Within the gastric antrum
Answer: c

Asir Surgery MCQs Bank. © 1422H-2002- first impression ©


195
This project was raised after an idia by Dr. Gharama Al-Shehri (consultant surgeon). Developed and typed by Dr. Ghazi Al-Shumrani (intern).
Esophagus, stomach & duodenum
Gastric ulcers can occur anywhere in the stomach, although they usually present on the lesser curvature near the incisura
angularis. About 60% are located at or slightly above the angularis. Fifteen percent to 23% of gastric ulcers are within the
gastric antrum and 10% are high on the lesser curvature. Only 5% of gastric ulcers are found on the greater curvature. In
addition, 97% of all gastric ulcers occur within 2 cm of the junctional zone between fundic and antral mucosa. Gastric
ulcers appear at different distances from the pyloric sphincter because the antrum extends for variable (2 to 16 cm) distances
from the pylorus. It is interesting that with increasing age this junctional zone moves proximally along the lesser curvature,
as does the incidence of gastric ulcer.

108.Which of the following statement(s) regarding gastric mucosal defense is/are correct?

a.Gastric mucus, produced by the surface epithelial cells, forms an unstirred layer over the gastric surface
b.Gastric parietal cells produce a bicarbonate-rich fluid
c.Production of gastric bicarbonate is stimulated by prostaglandins and inhibited by non-steroidal antiinflammatory
drugs (NSAIDs)
d.Gastric mucus provides substantial buffering capacity that maintains near-neutrality near the epithelial surface
Answer: a, c

Gastric mucus is produced by the surface epithelial cell. In addition to serving as a lubricant, mucus also enhances mucosal
defense by forming an unstirred layer overlying the epithelial surface. In humans, the thickness of this gel layer has been
reported to be about 500 µm. The vacuoles containing mucus in the cytosol of the surface epithelial cell are released by
cholinergic stimulation and prostaglandins. Synthesis and release are inhibited by aspirin-like compounds that inhibit
cyclooxygenase. Gel mucus retards proton mobility by a factor of only three or four, a degree that is inadequate to maintain
a near neutral pH at the apical membrane of the surface epithelial cell.
In addition to producing mucus, the gastric surface epithelial cells secrete a bicarbonate-rich fluid. The amount of
bicarbonate secreted is about 5% to 10% of the amount of acid that the same surface of mucosa can produce. As with
mucus, bicarbonate secretion is stimulated by cholinergic agents and prostaglandins and inhibited by cyclooxygenase
inhibitors.
The release of bicarbonate into the gel mucus provides a significant mucosal defense by maintaining a near neutral acid-
base milieu at the apical membrane of the surface epithelial cells. When the luminal pH is around 3.0, the apical membrane
of the surface epithelial cell may be exposed to a pH of about 5.0.

109.A 35-year-old smoker is involved in a house fire and receives a 45% total surface area burn. One half of the burned
surface appears to be third degree. On the third post-burn day, the patient is noted to have bloody drainage from a
nasogastric tube and a decrease of 5% in his hematocrit. Appropriate management should include which of the
following?

a.Urgent upper gastrointestinal contrast study to delineate site of bleeding


b.Immediate selective arteriography via the left gastric artery to diagnose and treat presumed stress ulceration
c.Urgent esophagogastroduodenoscopy to diagnosis the cause of bleeding
d.Urgent intravenous infusion of vasopressin at 0.2–0.4 IU/min
Answer: c

Patients who have sustained a major thermal burn of 35% or more of their body surface area are at a predictably high risk
for the development of gastric erosions and hemorrhage. Endoscopy has demonstrated that gastric erosions are present in
93% of these patients, whereas the occurrence of severe acute upper gastrointestinal hemorrhage in severely burned patients
ranges between 25% to 50%.
At least 60% of patients at risk develop stress erosions within 1 to 2 days after the precipitating event. Painless upper
gastrointestinal bleeding may be the only clinical sign. The onset of hemorrhage is often delayed, usually occurring 3 to 10
days after the onset of the primary disease.
Esophagogastroduodenoscopy is the diagnostic modality of choice to confirm the diagnosis and to differentiate stress
erosion from other sources of upper gastrointestinal hemorrhage. Correct identification of the bleeding source is made in
greater than 90% of instances. If endoscopy is not diagnostic, visceral angiography through selective catheterization of the
left gastric or splenic vessels may provide information regarding the primary vessel supplying the bleeding site. In contrast,
barium examinations are usually of little value, due to the superficial nature of stress erosions, and in fact may be
detrimental by interfering with the interpretation of subsequent arteriography.

Asir Surgery MCQs Bank. © 1422H-2002- first impression ©


196
This project was raised after an idia by Dr. Gharama Al-Shehri (consultant surgeon). Developed and typed by Dr. Ghazi Al-Shumrani (intern).
Esophagus, stomach & duodenum

110.Agents demonstrated to have an efficacy of greater than 90% for prophylactic treatment of stress ulceration include
which of the following?

a.Antacids
b.H2 receptor antagonists
c.Sucralfate
d.Misoprostil
Answer: a, b, c

The hourly administration of antacid (30 to 60 mL) by nasogastric tube, maintaining the gastric luminal fluid at pH above
3.5, has proven to be effective prophylaxis. In a study of 100 seriously ill patients who were randomly assigned to receive
placebo or antacid prophylaxis, bleeding was detected in 25% of patients given no prophylaxis, compared to 4% of patients
given antacids through the nasogastric tube. In a review of data derived from 16 prospective trials, when overt bleeding
manifested by melena, hematemesis, or transfusion requirement was used as the minimum criterion, there was no significant
difference in risk of bleeding, comparing antacids and cimetidine.
Continuous infusions of any of the H2-receptor antagonists provides more consistent maintenance of an intraluminal gastric
pH of greater than 3.5 than do the standard intermittent-infusion regimens. Advantages of continuous infusion of these
agents include a potential reduction in toxicity, decreased pharmacy costs and nursing duties, and possible enhancement of
therapeutic benefit. Controlled trials suggest that sucralfate, 1 g every 6 hours, may be as effective as antacids or cimetidine
prophylactically. In 100 critically ill patients, bleeding occurred in 6% of patients receiving antacids or cimetidine, while
none of the 34 patients on sucralfate bled.
Given exogenously, natural or synthetic prostaglandins of the E, F, and I series inhibit gastric acid secretion. One group
compared the efficacy of 15(R)-15 dimethyl PGE2 given at antisecretory doses to antacids and found that stress-related
bleeding occurred in 50% of patients given the synthetic prostaglandin derivative, compared to only 14% of patients
receiving antacids.

111.A 45-year-old male presents with symptoms of epigastric pain, worsened with ingestion of food. Physical
examination is normal. Upper abdominal ultrasonography is unremarkable. Contrast radiography reveals a 2 cm ulcer
in the gastric fundus along the lesser curvature. Therapy with omeprazole 20 mg per day is begun but symptoms persist
3 weeks later. Appropriate management includes which of the following?

a.Increase in omeprazole dose to 40 mg per day


b.Addition of sucralfate 1 gm every 8 hours
c.Addition of cimetidine 200 mg b.i.d.
d.Esophagogastroduodenoscopy with biopsy of ulceration
Answer: d

About 5% of ulcers appearing radiographically benign are malignant} Gastroscopy is the most reliable method of
distinguishing benign and malignant gastric ulcer, with an accuracy of more than 97% if multiple biopsies and brushings for
cytology are performed. Clinical features prompting early endoscopic evaluation include major weight loss, symptoms of
gastric outlet obstruction, a palpable abdominal mass, and stool hemoccult positivity or blood loss anemia. Endoscopic
features that suggest malignancy include an exophytic mass, abnormal or disrupted mucosal folds, necrotic ulcer crater,
bleeding from the edge of the ulcer crater, a stepwise depression of the ulcer edge, heaped-up margins, or small extensions
of the ulcer that blur a portion of the ulcer wall. If initial biopsies do not demonstrate malignant cells but the endoscopic
appearance strongly suggests that underlying the ulcer is a carcinoma, repeat endoscopy with deeper biopsies should be
undertaken.

112.A 52-year-old woman is hospitalized with acute upper gastrointestinal hemorrhage. Endoscopic examination reveal
a 2.5 cm ulcer in the area of the incisura angularis. The remainder of the endoscopic examination is normal. Continued
bleeding requires operative therapy. Optimal therapy consists of which of the following?

a.Gastrotomy with oversewing of the bleeding site


b.Distal gastrectomy including the area of ulceration
c.Proximal gastric vagotomy and oversewing of the bleeding ulcer
d.Truncal vagotomy, pyloroplasty, and oversewing of the bleeding ulcer
Asir Surgery MCQs Bank. © 1422H-2002- first impression ©
197
This project was raised after an idia by Dr. Gharama Al-Shehri (consultant surgeon). Developed and typed by Dr. Ghazi Al-Shumrani (intern).
Esophagus, stomach & duodenum
Answer: b

A distinction should be made among the different types of gastric ulcer in selecting the most appropriate operative
procedure, because treatment varies according to location, coexistent duodenal ulcer disease, and acid secretory status. The
elective operation of choice for a type I benign gastric ulcer is a distal gastrectomy with gastroduodenal anastomosis.
Gastrojejunostomy is an acceptable alternative. The ulcer should be included in the antrectomy specimen. The operative
mortality rate associated with this procedure is 2% to 3%, the recurrence rate is 3%, and a good to excellent clinical result
can be anticipated in more than 90% of patients. The addition of truncal vagotomy does not appear to diminish the
recurrence rate.
Definitive treatment for hemorrhage is accomplished by a procedure designed to control bleeding in addition to preventing
recurrent ulceration. An antrectomy, which includes the ulcer with gastroduodenostomy, is considered the procedure of
choice for surgical treatment of this complication. The quoted operative mortality rates in this setting range from 10% to
40%.

113.A 25-year-old man is involved in an automobile accident with resultant injuries including bilateral closed femur
fractures, left pulmonary contusion, and closed head injury. On post-injury day 4, significant upper gastrointestinal
hemorrhage begins. Endoscopic examination reveals an area of confluent ulceration with bleeding in the gastric fundus.
Endoscopic hemostasis fails. Appropriate immediate management includes:

a.Lavage of gastric contents with iced saline


b.Urgent total gastrectomy
c.Selective arterial infusion of vasopressin via the left gastric artery
d.Insertion of Sangstaken-Blakemore balloon
Answer: c

Initial efforts to control gastric hemorrhage consist of gastric lavage using warmed saline. Lavage serves to fragment
existing clots and to remove any pooled blood, reducing fibrinolysis at bleeding sites. Over 80% of patients who present
with upper gastrointestinal hemorrhage stop bleeding using this approach. Definitive treatment of ongoing acute active
stress bleeding by antacids is largely unsuccessful. Administration of H2-receptor blocking agents once active
gastrointestinal bleeding has commenced is also usually ineffective as a definitive form of therapy.
The endoscope has become the preferred therapeutic as well as diagnostic instrument with electrocautery and laser
photocoagulation capabilities. If endoscopic therapy fails, angiography offers an additional means for the control of
bleeding by selective infusion of vasopressin into the splanchnic circulation via the left gastric artery. Vasopressin is
administered by continuous infusion through the catheter at a rate of 0.2 to 0.4 IU/min for a maximum of 48 to 72 hours.
About 10% to 20% of patients with acute stress ulcers continue to bleed or have recurrent bleeding despite these measures.
In these patients, total gastrectomy has a mortality ranging from 17% to 100%. In general, operative mortality rates for
acute stress-induced hemorrhage range from 30% to 60% regardless of the surgical procedure undertaken.

114.Which of the following statements regarding gastroplasty and gastric bypass for morbid obesity is/are correct?

a.Horizontal gastroplasty techniques that rely on a single horizontal application of a stapling device are associated with
weight loss “failure” rates of 40% to 70%
b.Gastric bypass is followed by progressive weight loss over a period of 36 months
c.Gastric bypass is associated with a “failure” of weight loss in 10 to 15% of patients
d.With three superimposed applications of a stapling device, gastric bypass staple line dehiscence occurs in less than
2%
Answer: a, c, d

Horizontal gastroplasties include a single application of a 90-mm stapling device without suture reinforcement of the
“stoma’’ between upper and lower gastric pouches or a double application of staples with either a central or lateral prolene-
reinforced stoma. The failure rates (loss of less than 40% excess weight) for horizontal gastroplasty procedures ranges from
40% to 70%. The vertical banded gastroplasty (VBGP) is a procedure in which a stapled opening is made in the stomach
with the stapling device 5 cm from the cardioesophageal junction. Two applications of a 90-mm stapling device are made
between this opening and the angle of His, and a 1.5 5 cm strip of polypropylene mesh is wrapped around the stoma on the
lesser curvature and sutured to itself.

Asir Surgery MCQs Bank. © 1422H-2002- first impression ©


198
This project was raised after an idia by Dr. Gharama Al-Shehri (consultant surgeon). Developed and typed by Dr. Ghazi Al-Shumrani (intern).
Esophagus, stomach & duodenum
Gastric bypass can be performed with placement of staples in a vertical or horizontal direction; the vertical direction is
preferred because there is less risk of gastric pouch devascularization or splenic injury. With three superimposed
applications of a 90-mm stapler, the incidence of staple line disruption has been less than 2%.
Roux-en-Y gastric bypass has significantly better weight loss than VBGP. Although 10% to 15% of patients fail gastric
bypass, weight loss seems to remain stable in most patients over 5 years or more after surgery.

115.Severe obesity is associated with a large number of associated problems that form the basis of the term morbid
obesity. Documented causes of excess mortality in severely obese patients include:

a.Coronary artery disease


b.Hypertension
c.Adult-onset diabetes mellitus
d.Obesity hypoventilation and sleep apnea
e.Pulmonary embolization
Answer: a, b, c, d, e

Morbid obesity is arbitrarily defined as 100 pounds above ideal body weight as defined by actuarial tables. Premature death
is much more common in the severely obese; morbidly obese men aged 25 to 34 years have a 12-fold increase in mortality
relative to normal. Causes of early mortality include coronary artery disease, hypertension, impaired ventricular function,
diabetes mellitus, sleep apnea and other hypoventilation syndromes, pulmonary embolization, and necrotizing soft tissue
infections.

116.Jejunoileal bypass was formerly performed as a weight reduction procedure. The operation has now been
abandoned due to the development of serious long-term complications associated with the procedure. Which of the
following statements correctly characterize results following jejunoileal bypass?

a.Kidney stones occur with increased frequency due to increased absorption of pyruvate from the colon
b.The most serious complication of jejunoileal bypass is development of cirrhosis due to protein calorie malnutrition
c.Bacterial overgrowth in the bypassed segment can be treated with oral vancomycin
d.Rapid weight loss following jejunoileal bypass is associated with development of gallstones
Answer: b, d

Jejunoileal bypass is associated with a number of early and late complications. Malabsorption of bile salts, coupled with
rapid weight loss, significantly increases risk of gallstone development. Multiple kidney stones result from excessive
absorption of oxylate from the colon where oxylate is ordinarily chelated with calcium. Malabsorption results in severe
diarrhea, electrolyte abnormalities, metabolic acidosis and anemia. Bacterial overgrowth in the bypassed intestinal segment
coupled with protein malabsorption is postulated to be responsible for development of cirrhosis, the most serious
complication of jejunoileal bypass. Bacterial overgrowth can be temporarily suppressed by metronidazole. Development of
hepatic dysfunction is an indication for reversal of the bypass.

117.Which of the following statements is correct with regard to gastric bypass for obesity?

a.Rapid weight loss following successful gastric bypass for obesity is associated with an increased risk of developing
cholelithiasis
b.Marginal ulcer develops in 25% of gastric bypass patients
c.Vitamin B12 deficiency is a potential complication of gastric bypass due to gastric mucosal atrophy
d.Anastamotic leak after gastric bypass is often heralded by bradycardia
Answer: a

The most serious complication after gastric bypass for obesity is anastamotic dehiscence. Leak is presumed to occur
secondary to gastric necrosis due to ischemia from staple line application or short gastric vessel ligation. Affected patients
may have little pain, with tachycardia, tachypnea and fever as the only manifestations. Physical examination in morbidly
obese patients with peritonitis is unreliable. Marginal ulcers occur in only 10% or less of gastric bypass patients and respond
to H2 receptor antagonists. Rarely, polyneuropathy has been noted after gastric bypass, usually in association with
intractable vomiting and protein calorie malnutrition. Vitamin B12 deficiency has been noted after gastric bypass due to
Asir Surgery MCQs Bank. © 1422H-2002- first impression ©
199
This project was raised after an idia by Dr. Gharama Al-Shehri (consultant surgeon). Developed and typed by Dr. Ghazi Al-Shumrani (intern).
Esophagus, stomach & duodenum
decreased acid digestion of vitamin B12 in food; monthly B12 supplementation should be routine. Cholelithiasis occurs in
about one third of morbidly obese patients and gallstone formation is accelerated in the early postoperative period by the
effects of rapid weight loss.

118.With regard to operative treatment of gastric carcinoma, which of the following statements is/are correct?

a.Resectional margins of 2 cm are necessary to prevent recurrence due to intramural metastases


b.Prophylactic splenectomy has been shown to improve outcome in similarly staged patients
c.Extended lymphadenectomy including nodes along the aorta and esophagus has not been shown to improve survival
in North American trials
d.Long-term survival is rare if adjacent organs must be resected to achieve local control
Answer: c, d

In gastric cancer, microscopic involvement of the resection margin by tumor cells is associated with poor prognosis. In
contrast to colon cancer, gastric cancer frequently demonstrates extensive intramural spread. Retrospective studies suggest
that a line of resection 6 cm from the tumor mass is necessary to ensure a low rate of anastamotic recurrence.
The value of extended lymphadenectomy in the treatment of gastric adenocarcinoma is controversial. The largest favorable
experience has been reported by Japanese surgeons, where retrospective studies have suggested an improvement of
approximately 10%, stage for stage, for patients with advanced disease. The benefits of extensive lymphadenectomy have
not been confirmed in western countries.
Histologically positive lymph nodes are frequently present in the splenic hilum and along the splenic artery, and routine
splenectomy has been practiced in some centers. Prophylactic splenectomy has not been demonstrated to improve outcome
for similarly staged patients. Resection of adjacent organs may be required for local control if direct invasion has occurred.
In this circumstance, operative morbidity is increased, and long-term survival is rare.

119.Which of the following statement(s) characterizing gastric lymphoma is/are correct?

a.More than half of gastrointestinal lymphomas occur in the stomach


b.The peak incidence of gastric lymphoma is in the 2nd and 3rd decade
c.Endoscopic biopsy is positive diagnostically in 90% of cases
d.Gastric perforation occurs in 40% of patients treated with cytolytic agents instead of gastrectomy
Answer: a, c

The stomach is the site of more than half of gastrointestinal lymphomas and is the most common organ involved in
extranodal lymphomas. Gastric lymphoma is distinctly uncommon in children and young adults. The peak incidence is in
the sixth and seventh decades. Radiologic findings are similar to those noted for adenocarcinoma. Endoscopic examination
has become the diagnostic method of choice. Endoscopic biopsy, combined with endoscopic brush cytology, provides
positive diagnosis in some 90% of cases. When gastric lymphoma is first diagnosed by endoscopic means, evidence of
systemic disease should be sought. CT of chest and abdomen (to detect lymphadenopathy), lymphangiography, bone
marrow biopsy, and biopsy of enlarged peripheral lymph nodes may all be appropriate.
A multimodality treatment program is used in most centers for primary gastric lymphomas, with gastrectomy as the first
step in the therapeutic strategy. Increasing numbers of patients are treated with chemoradiation therapy alone. The risk of
hemorrhage or perforation was frequently alluded to in the past and has probably been overstated. The risk of perforation in
primary gastric lymphomas that are treated with cytolytic agents in unresected patients approximates 5%.

120.A patient with gastric adenocarcinoma undergoes subtotal gastrectomy. Pathological examination reveals that the
tumor penetrates to the serosa. Regional lymph nodes are not involved. Distant metastases are not detected. What is the
correct tumor stage and 5-year survival rate?

a.Stage I: 90% 5-year survival


b.Stage II: 45% 5-year survival
c.Stage III: 15% 5-year survival
d.Stage II: 15% 5-year survival
e.Stage III: 2% 5-year survival

Asir Surgery MCQs Bank. © 1422H-2002- first impression ©


200
This project was raised after an idia by Dr. Gharama Al-Shehri (consultant surgeon). Developed and typed by Dr. Ghazi Al-Shumrani (intern).
Esophagus, stomach & duodenum
Answer: b

For early lesions of the antrum or middle stomach, distal subtotal gastrectomy including 80% of the stomach provides
satisfactory 5-year survival without increases in operative morbidity. Proximal gastric lesions or larger middle stomach
lesions may require total gastrectomy or esophagogastrectomy to encompass the tumor. Regardless of the extent of gastric
resection, patients with more advanced tumors fare poorly because of the increased likelihood of lymphatic and
hematogenous spread.
The TNM system is shown in Table 25-2. Five-year survival for stage I patients (in situ carcinoma) is close to that of the
normal population. In stage II, 5-year survival approximates 45%, while 15% of stage III patients survive 5 years.
Prolonged survival with systemic metastases in negligible.

121.Which of the following conditions is considered to increase the risk of gastric cancer?

a.Pernicious anemia
b.Prior partial gastrectomy
c.Gastric hyperplastic polyps
d.Gastric adenomatous polyps
Answer: a, b, d

The risk of developing gastric cancer is greater in stomachs that harbor adenomatous polyps. The risk of developing cancer
has been estimated at 10% to 20% and is greatest for polyps more than 2 cm in diameter. Hyperplastic polyps, while
common in the normal population, do not have malignant potential. Gastric malignancy is increased in individuals with
chronic gastritis associated with pernicious anemia. When pernicious anemia has been present for 5 years, the risk of gastric
malignancy is twice that of age-matched controls. An increased risk of gastric carcinoids also exists in patients with
pernicious anemia, presumably due to the effects of long-standing hypergastrinemia. A 3-fold increased risk of gastric
cancer also exists in patients that have previously undergone partial gastric resection. Postgastrectomy cancer is a long-term
concern with increased incidence of malignancy not observed until 15 years postoperatively.

122.A 55-year-old male is evaluated because of symptoms of epigastric pain and anorexia. Physical examination is
normal except for guaiac positivity of stool. Upper endoscopic examination reveals a 1.5 cm ulcer along the lesser
curvature of the stomach proximal to the incisura angularis. Optimal management consists of which of the following:

a.Sucralfate 1 gm q.i.d. for 8 weeks


b.Endoscopic biopsy of the ulcer rim
c.Endoscopic cautery of the ulcer base
d.Endoscopic biopsy of the ulcer base
e.Misoprostol 400 mg b.i.d. for 8 weeks
Answer: b

The symptoms produced by gastric cancer and benign gastric ulcer are non-specific and often similar. Pain is present in 70%
of patients with gastric cancer and is usually constant, non-radiating, and not improved by food ingestion. Physical
examination is usually normal in patients with early gastric cancer. Guaiac positivity is noted in one third.
Fiberoptic endoscopy is the definitive diagnostic method. Although the endoscopic appearance of gastric ulcers may suggest
benign or malignant origins, definite distinction can only be made by gastric biopsy. Accurate diagnosis of gastric cancer
can be made in 95% of cases if multiple biopsies are obtained from the ulcer rim. Biopsy of the ulcer base will more
frequently reveal necrotic material.

123.Which of the following statement(s) regarding gastric leiomyosarcoma is/are correct?

a.Leiomyosarcomas occur with peak frequency in the 2nd and 3rd decades
b.The primary histological indicator of aggressive behavior is the number of mitoses per microscopic field
c.Leiomyosarcomas are usually radiosensitive
d.Lymphadectomy is not indicated during resection because metastases are usually hematogenous
Answer: b, d

Asir Surgery MCQs Bank. © 1422H-2002- first impression ©


201
This project was raised after an idia by Dr. Gharama Al-Shehri (consultant surgeon). Developed and typed by Dr. Ghazi Al-Shumrani (intern).
Esophagus, stomach & duodenum
Leiomyosarcomas occur with equal frequency in both sexes in the sixth and seventh decades of life. The tumor frequently
may have prominent extraluminal growth and attain large size before causing symptoms. Leiomyosarcomas must be
differentiated from their benign counterparts, leiomyomas. Grossly, the tumors are firm, gray-white masses; a
pseudocapsule separating tumor from normal smooth muscle may occasionally be present. When the tumors reach a large
size, central necrosis is common. Leiomyosarcomas are usually graded histologically, with the frequency of mitotic figures
the prime indicator of aggressive behavior. Lesions with more than 5 to 10 mitoses per 10 high-power fields demonstrate
increased metastasis.
Intraperitoneal sarcomatosis is frequent, as is local recurrence after resection. Metastasis occurs by way of the
hematogenous route, and thus hepatic involvement is common. Lymphatic metastasis is observed in less than 10% of
patients. Negative surgical margins must be ensured histologically, but lymphadenectomy is not indicated because of the
low frequency of lymphatic metastasis. Leiomyosarcomas are not radiosensitive, and chemotherapy has not been shown to
improve survival.

Asir Surgery MCQs Bank. © 1422H-2002- first impression ©


202
This project was raised after an idia by Dr. Gharama Al-Shehri (consultant surgeon). Developed and typed by Dr. Ghazi Al-Shumrani (intern).

You might also like